Минимальная величина тока опасная для жизни человека: Какой ток опасный для человека – постоянный или переменный, и почему?

Содержание

Опасный ток для человека. Величина, виды воздействий

В быту и на производстве мы сталкиваемся с различными электроприборами, электроустановками. Соблюдая правила электробезопасности и обладая знаниями в данной сфере можно уменьшить вероятность попадания под опасное воздействие электрического тока и напряжения.

В данном вопросе объединяются знания инженерного и медицинского характера, применение которых в комплексе, увеличит результат по снижению уровня электротравм дома и на производстве.

Действие электрического тока на организм человека

Ток, в отличие от других опасных сред, не обладает цветом, запахом, невидим.

Электрический ток оказывает следующие виды воздействия на организм человека: термическое, электролитическое, биологическое. Рассмотрим каждое из этих воздействий более подробно.

Термическое воздействие заключается в ожогах участков тела, нагреве сосудов и нервных окончаний. Этот вид действия называют еще тепловым. Потому что тепловая энергия, полученная из электрической образует ожоги.

Электролитическое воздействие приводит к разложению крови и других жидкостей в организме посредством процесса электролиза, что вызывает нарушения в физико-химическом составе этих жидкостей. Суть повреждений сводится к молекулярному уровню – загустевание крови, изменение заряда белков, паро- и газообразование в организме.

Биологическое воздействие электротока на организм сопровождается раздражением и возбуждением органов. Это вызывает судороги, сокращения.

В случае с сердцем и легкими это воздействие может привести к летальному исходу по причине прекращения деятельности органов дыхания и сердца.

Биологическое воздействие вызывает механические повреждения органов, суставов человека. Также механические повреждения может вызвать падение человека с высоты из-за воздействия электрического тока.

Опасная, безопасная и смертельная сила тока для человека

Нельзя считать какую-либо величину тока безопасной для человека. Существует лишь более и менее опасная величина электротока. Каждый человек имеет внутреннее сопротивление, на величину которого влияет множество факторов (толщина кожи, влажность помещения и тела человека, путь протекания тока).

Самым опасным путем протекания тока является направление нога-голова, рука-голова, так как при этом путь идет через сердце, мозг, органы дыхания. А большая величина тока может вызвать остановку сердца и остановку дыхания. Именно эти причины являются наиболее вероятными причинами летальных исходов при протекании электротока.

Считается, что постоянный ток более безопасный, чем переменный в сетях до 500В. При напряжении выше 500 вольт опасность постоянного тока возрастает.

Частота сети влияет на степень тяжести электротравмы. Промышленная частота в 50 Гц является более опасной, чем частота в 500Гц. При высокой частоте наблюдается так называемый «скин-эффект», когда ток проходит не по всему проводнику, а лишь по его поверхности. А значит, внутренние органы напрямую не затрагиваются.

Также на степень опасности воздействия тока на человека влияет продолжительность нахождения человека под воздействием тока. Здесь зависимость линейная – чем дольше, тем больше разрушений и неблагоприятных последствий.

Приведем пороговые значения переменного и постоянного тока и возможные реакции организма на эти воздействия:

Проходя через человеческое тело, ток может создавать электрические травмы или электрические удары.

Электрический удар подразумевает, что ток возбуждает ткани организма, что вызывает их сокращение и судороги. Существует 4 группы электроударов: судороги, судороги с потерей сознания, потеря сознания с нарушением дыхания и работы сердца, клиническая смерть.

При электрической травме ток наносит прямые повреждения тканям и органам человека. Это могут быть электрические ожоги, металлизация кожи, электрические метки и механические повреждения.

Электрические ожоги бывают токовыми и дуговыми. Действие токового ожога связано с прохождением тока через тело человека. Дуговой ожог возникает между человеком и проводником электротока высокого напряжения, вследствие возникновения дуги между ними. Температура дуги может достигать тысяч градусов по Цельсию. Такой ожог гораздо опаснее и может плюс ко всему сопровождаться возгоранием одежды пострадавшего.

Металлизация кожи происходит, когда под действием тока в кожу попадают частицы металла, при этом проводимость кожи увеличивается, что повышает травмоопасность.

Электрические метки – это места, через которые ток входит и выходит из тела человека. Наиболее часто встречаются на ногах и руках.

В любом случае следует стараться избегать касания токоведущих частей проводящими предметами (ловить рыбу под ЛЭП, нести стремянку вблизи шин напряжения), не использовать провода и кабели с ослабленной изоляцией, соблюдать правила безопасности при нахождении и работе в электроустановках. Берегите здоровье себя и своих родных.

Сила тока, смертельная для человека

    Смертельным для человека является ток силой 0,1 а и выше. Ток силой 0,05—0,10 а очень опасен, при воздействии на человека вызывает обморочное состояние уже нри силе тока 0,03 а человек не может отор- 
[c.339]

    В связи с этим в ряде случаев даже ток осветительной сети может оказаться смертельным для человека, так как сила тока при прохождении через тело человека может достигнуть (согласно закону Ома) [c.137]

    Переменный ток оказывает более сильное действие, чем постоянный. Применяемый в промышленности переменный ТОК средней частоты представляет для человека определенную опасность уже при силе тока 0,01 А, а поражение током силой 0,1 А и более приводит к смертельному исходу. [c.202]


    В сухих помещениях опасным для человека считается напряжение выше 36 В. Смертельной является сила тока 0,1 А, а ток 0,05 А вызывает судорожное сокращение мышц, не позволяющее человеку оторваться от источника поражающего напряжения. 
[c.103]

    Действие статического электричества на человека смертельной опасности не представляет, поскольку сила тока составляет небольшую величину. Искровый разряд статического электричества человек ощущает как тол- [c.104]

    Опасным для человека является переменный ток промышленной частоты более 15 мА, при котором человек не может самостоятельно освободиться от источника тока. Ток в 50 мА вызывает тяжелое поражение, а ток в 100 мА, воздействующий более 1—2 с, является смертельно опасным. При поражении человека постоянным током опасной считается сила тока 20—25 мА, так как пострадавший не может самостоятельно освободиться от источника тока. 

[c.34]

    Ток такой силы для человека является смертельно опасным. [c.14]

    Действие статического электричества на человека смертельной опасности не представляет, поскольку сила тока невелика. Искровой разряд статического электричества человек ощущает как толчок или судорогу. При внезапном уколе возможен испуг и вследствие рефлекторных движений человек может сделать непроизвольно движения, приводящие к падению с высоты, попаданию в неогражденные части машин и др. Имеются также сведения, что длительное воздействие статического электричества неблагоприятно отражается на здоровье работающего, на его психофизиологическом состоянии. Вредно влияет на состояние человека также электрическое поле, возникающее при статической электризации 

[c.192]

    Согласно закону Ома, при расчетном сопротивлении тела человека 1000 Ом и напряжении осветительной сети 220 В сила тока составит 220 мА, т. е. при такой силе тока возможен смертельный исход. [c.41]

    Наиболее опасным является переменный ток низкой частоты (в том числе частотой 50 Гц). При силе переменного тока до 0,015 А опасности для человека нет, но уже при силе более 0,015 А возможны тяжелые последствия. За величину отпускающей силы тока принята величина 0,01 А, токи силой 0,09—0,1 А и выше являются смертельными. 

[c.77]

    Степень тяжести поражения определяется величиной тока, протекающего через тело человека. Ток силой 0,05 а является уже опасным, а ток силой 0,1 а — смертельным. [c.34]

    Ток такой силы смертельно опасен для человека. [c.16]

    Сила электрического тока, проходящего через тело человека, является основным фактором, определяющим исход поражения. Человек ощущает действие переменного тока промышленной частоты при его величине около 1 мА. При такой силе тока появляется раздражение чувствительных нервных окончаний в местах прикосновения к токоведущей части. При силе тока 8—10 мА раздражение распространяется более глубоко, но человек может самостоятельно освободиться от действия тока при силе тока 10—15 мА возникает локальная судорога и человек не может разжать пальцы руки, в которой зажата токоведущая часть. При силе тока 25—50 мА и частоте 50 Гц, помимо судорожного сокращения мышц конечностей, возникают судороги дыхательных мышц, в результате которы может наступить смерть от удушья. Сила тока 100 мА и более считается смертельной. При такой силе тока и частоте 50—60 Гц происходит беспорядочное сокращение сердечных мышц (фибрилляция сердца). Кратковременное (до 1—2 с) действие больших токов (более 5 А) не вызывает фибрилляции сердца. При такой силе тока сердечная мышца резко сокращается и остается в таком состоянии до отключения тока, после чего продолжает работать. 

[c.11]


    Следует всегда помнить, что действие электрического тока на человеческий организм зависит от многих факторов. Большое значение при этом имеет частота тока, время прохождения его через тело человека, величина участка пораженного тела, а также состояние организма человека. В настоящее время установлено, что прохождение электрического тока силой более 100 мА через тело человека, как правило, приводит к смертельному исходу. Ток силой 50—100 мА вызывает потерю сознания, а менее 50 мА — сокращение мышц, так что иногда пострадавший не в состоянии разжать руки и освободиться от токонесущих поверхностей самостоятельно. [c.9]

    Электрический ток силой более 0,1 а при напряжении до 1000 в представляет, как правило, смертельную опасность для человека. Если человеку в этом случае не оказать немедленную помощь, то спустя 6—8 мин его уже нельзя будет спасти. При поражении электрическим током нарушается деятельность жизненно важных центров и органов человека центральной нервной системы, сердечнососудистой системы и дыхания. [c.286]

    Электрический ток, проходя через тело человека, может вызвать тяжелые травмы, а иногда и смерть. Степень поражения электрическим током определяется его силой, характером пути прохождения тока через тело человека, длительностью его прохождения, его частотой и индивидуальными свойствами человека. Наиболее опасен ток промышленной частоты. Токи высокой частоты не вызывают электрического шока, но при длительном прохождении могут привести к чрезмерному нагреванию илн ожогу отдельных частей тела. При силе тока промышленной частоты 0,05 А, проходящего через человека, возможен смертельный исход, а при силе тока 0,1 Л и более неизбежен смертельный исход. Наиболее опасные поражения возникают при прохождении тока через сердце и мозг. [c.461]

    Электрофоретическое оборудование обычно работает во влажной атмосфере, причем величины напряжения и силы тока, как правило, превышают безопасные пределы. Неправильное обращение с приборами уже привело к нескольким несчастным случаям со смертельным исходом. Омическое сопротивление человеческого тела, обычно составляющее 10 —10″ Ом, существенно зависит от физиологического состояния человека и влажности кожи. Для человека опасен даже ток силой 10 мА, так как при поражении током пострадавший обычно не может сам отсоединиться от проводника. Ток силой более 25 мА вызывает серьезные повреждения в организме —остановку сердца, паралич дыхательных мышц, ожоги и т. д., которые могут привести к смерти. Учитывая, что сопротивление тела 10 Ом, напряжение всего лишь в 100 В способно привести к несчастному случаю в результате уменьшения сопротивления вследствие шока, сопровождающегося потоотделением и (или) повреждением кожи, опасно даже меньшее напряжение. Таким образом, приборы для электрофореза и изоэлектрического фокусирования, являющиеся источниками электрического тока, могут представлять опасность для жизни. Если источники питания стабилизованы, то опасность возрастает, так как напряжение во время разъединения проводов или разрыва проводящих соединений в электрофоретической камере увеличивается. При работе на приборе для дискретного электрофореза в полиакриламидном геле, который обычно снабжен стабилизованным источником питания, риск часто недооценивают. [c.327]

    Опасным для организма человека является ток силой более 15 мА, при котором трудно самостоятельно оторваться от электродов, и смертельным — 100 мА и более. [c.206]

    Высокое напряжение. Наибольшую опасность представляют искровые генераторы, дающие на выходе напряжение до 20 кв при довольно большой мощности. Разряд конденсаторов колебательного контура, заряженных до этого напряжения, через человека может привести к смертельному исходу. Генераторы, выпускаемые промышленностью (например, вся серия генераторов ИГ), снабжены целым рядом защитных устройств дверцы шкафа, в котором расположены все приборы, имеют блокировку, отключающую питание при открывании шкафа вывод сделан специальным высоковольтным кабелем, корпус снабжен клеммой для заземления. При работе следует строго соблюдать правила обращения, предусмотренные инструкцией, в частности не включать генератор, не присоединенный к хорошему заземлению. Ни в коем случае нельзя для заземления пользоваться трубами водопроводной и отопительной систем. Если лаборатория не оборудована специальными заземленными шинами, то заземление нужно сделать, руководствуясь разработанными для этого правилами техники безопасности при работе с высоким напряжением. Этими же правилами следует руководствоваться при проектировании и эксплуатации нестандартных высоковольтных генераторов, монтируемых для тех или иных задач силами лаборатории. Применение ограждений из заземленных металлических сеток, специального высоковольтного кабеля, устройство блокировок, отключающих питающее напрян и разряжающих конденсаторы,— все эти меры должны неукоснительно соблюдаться. Меньшую опасность представляют источники высокочастотного напряжения для питания газоразрядных трубок, несмотря на то что напряжение соответствующих генераторов достигает 3—5 кв. Замыкание такого генератора через тело обычно никаких вредных последствий, кроме легкого кожного ожога, не дает. Это объясняется скин-эффектом — распространением высокочастотного тока только в тонком поверхностном слое проводника. Наоборот, источники постоянного тока напряжением около 1000 б, применяемые, например, для питания трубок с полым катодом, представляют довольно значительную опасность. Правда, мощность этих источников обычно невелика, что снижает их опасность, если в высоковольтную цепь не включены конденсаторы большой емкости. [c.50]


    В трехфазной сети с глухозаземленной нейтралью (рис. 1,в) сила тока, проходящего через человека, определяется фазным напряжением, сопротивлением тела человека и сопротивлением заземления нейтрали / о-Так как чел больше Яо, в этом случае опасность поражения человека электрическим током увеличивается по сравнению с опасностью в предыдущем случае. Однако при однофазном прикосновении, когда другая фаза замыкается на землю (аварийный режим), человек оказывается под полным линейным напряжением, и сила тока может оказаться смертельной. [c.44]

    Степень опасности от электрического удара зависит от силы тока, протекающего через тело человека. Сила тока в свою очередь зависит от величины приложенного напряжения и от сопротивления человеческого тела, на которое сильно влияет загрязненность и влажность кожи. Сопротивление человеческого тела колеблется от нескольких дe яtкoв тысяч до нескольких сотен омов. Поэтому при неблагоприятном случае напряжение в несколько десятков вольт может оказаться опасным. На одном из магниевых заводов был случай со смертельным исходом от напряжения 60 в. Имеет значение продолжительность воздействия тока на организм человека, частота переменного тока и индивидуальные особенности организма. [c.232]

    Можно ли считать, что протекание тока силой ме нее 6 мА через организм человека вполне безопасно Ни в коем случае Пороговые значения неотпускающе го тока определяются экспериментально — при этом испытуемый держит электрод в руке На практике элек трическая цепь далеко не всегда возникает по схеме ладонь—ладонь или ладонь—ноги Вполне вероятны и в действительности происходят поражения при ко topыx ток проходит через тыльную часть руки, пред плечье или голень В то же время на теле человека, в том числе на тыльной части рук, имеются чувствитель ные к току (активные) места Образование электриче ских цепей с участием этих уязвимых мест, приводит к тяжелым поражениям и смерти даже при очень ма лых токах Важно что смерть наступает и в тех слу чаях когда путь тока не лежит через жизненно важные органы — сердце, легкие мозг Зарегистрированы по ражения со смертельным исходом при напряжении 220 В и ниже, когда с токоведущими частями сопри касалась только одна рука и путь тока проходил от тыльной стороны руки к ладойи или даже с одной сто роны пальца на другую [32] [c.99]


Смертельный ток для человека в Амперах, какая величина

Современная жизнь очень тесно связана с электричеством. Постепенно арсенал домашних электроприборов все больше увеличивается. Некоторые решаются сами проводить установку оборудования, проводить электропроводку или ремонтировать электрооборудование.

Все это сопряжено с тесным контактом человека и тока. Незнание элементарных правил обращения с электричеством может привести к травме или даже смерти. Далее узнаем, какой смертельный ток для человека, что из себя представляет ток, какие травмы он может принести и некоторые другие вопросы.

В чем опасность удара электрическим током

Иногда важно знать не то, какая сила тока может убить человека, а реакцию человека и внешнюю обстановку. Как правило, для человека получение удара от электрического тока происходит неожиданно. В силу этого человек может делать непроизвольные движения и необдуманные поступки.

Например, стоя на стремянке и получив удар током, человек может потерять равновесие и упасть с высоты и получить серьезные травмы. Неслучайно в правилах по технике безопасности приводится множество правил, как правильно работать с электроприборами.

Смертельная сила тока для человека определяется продолжительностью воздействия, чем больше продолжительность, тем большие травмы наносятся телу.

Находясь под действием тока, человек может испытывать болезненные ощущения, что может привести к шоку. Могут обостриться хронические заболевания или появиться новые. При более серьезной травме возможна временная, длительная или постоянная потеря трудоспособности.

Действие тока опасно еще и тем, что он действует на работу сердца и легких, в тяжелых случаях полностью останавливая их работу. Какая сила тока смертельна для человека, определяется путями прохождения электрического тока.

Опасные пути прохождения электрического тока через тело

Если рассматривать статистику, то около 40% ток поражает человека через руки. При этом через сердце проходит 3,3% от общего тока. В этом случае смертельный ток для человека повышается, увеличивая его шанс к выживанию.

На втором месте идет поражение через правую руку в одну или обе ноги. Поскольку большинство людей правши, то показатель составляет 20%.
Процентное соотношение тока, проходящего через сердце, увеличивается более чем в два раза и достигает 6,7%. Значение смертельной силы тока для человека резко понижается, увеличивая шанс тяжелых травм или смерти.

Левшам, или людям, коснувшимся левой рукой находящейся под напряжением цепи, достается 17%. В этом случае через сердце проходит 3,7%, увеличивая их шанс на благополучный исход.

Самым безопасным является путь тока через ноги. Сердцу достается всего 0,4% от общего потока. Но такое поражение сравнительно редко, ему подвержены только 6% от общего числа всех пострадавших.

Самым тяжелым случаем является путь тока через голову. Если цепь соединяется через голову и ноги, то через сердечную мышцу проходит 6,8% всей силы тока. К счастью, таких случаев только 5%. Однако если цепь состоит из головы и рук, то на сердце обрушивается максимальный поток, составляющий 7%. Таких случаев зафиксировано 4%.

Виды электрических травм

Все травмы, полученные от поражения электрическим током, можно разделить на четыре вида:

  1. термические;
  2. электролитические;
  3. механические;
  4. биологические.

Термическое воздействие. Тело человека состоит примерно из 80% воды, в которой растворены соли и минералы или находятся во взвешенном состоянии другие элементы. Это делает воду электролитом, который довольно хорошо проводит электричество, а оно, в свою очередь, производит работу, то есть нагревает все тело. Это происходит при малых токах и длительном воздействии. При больших токах происходит выгорание тканей на пути прохода электричества.

Под электролитическим подразумевается распад жидкости (крови, лимфы), из-за чего она уже не может выполнять свои функции.

К механическим относятся: разрыв кровеносных сосудов из-за давления пара, обрыв сухожилий и перелом костей из-за сокращения мышц.

Биологические нарушения – это нарушение кровообращения, дыхания и других органов. Для того чтобы понять, ток какой силы смертельно опасен для человека, следует учесть сопротивление тела человека.

Сопротивление человека и от чего оно зависит

Сопротивление тела человека чисто индивидуально и может сильно отличаться между индивидуумами. Складывается оно из сопротивления эпидермиса – наружного покрова и внутренних органов.

Чтобы вывести таблицы и схемы это значение условно принимается за 1 000 Ом или 1 кОм. Однако, это правило справедливо при непосредственном контакте тела.

Если ток проходит через ноги, сопротивление складывается из сопротивления тела, одежды, обуви и поверхности, на которой стоит человек. Поэтому если в первом случае смертельный ток для человека имеет одно значение, то во втором оно будет совершенно другим.

Кроме того, на сопротивление человека влияет множество других факторов. Например, здоровые сильные люди обладают большим сопротивлением, чем больные и слабые.

Вспотевшее тело уменьшает сопротивление, это же происходит, если человек возбужден или находится в подавленном состоянии. Поэтому очень сложно определить, какой ток будет проходить при тех или иных условиях. Тем не менее теоретически определено, каким будет смертельный ток для человека в амперах.

Какая величина тока считается смертельной для человека

Сила тока в 1 А — очень большая величина, поэтому чтобы определить смертельный ток для человека, используют меньшую величину – миллиамперы, мА. В 1 А содержится 1 000 мА.

Стоит уточнить, что смертельным ток становится не только из-за действия на органы, но и неспособности человека самостоятельно освободиться от действия электричества.

Так, при переменном токе силой 10–15 мА человек уже не может самостоятельно разжать пальцы рук и, продолжая находиться под действием тока, он подвергается смертельной угрозе. Для постоянного тока это значение составляет 50–80 мА.

При этом отмечаются четыре последствия воздействия тока:

  • без потери сознания;
  • с потерей сознания;
  • клиническая смерть;
  • биологическая смерть.

Находясь в сознании, человек еще может рассуждать и позвать на помощь, что увеличивает его шанс на выживание и получение наименьшего ущерба.

При потере сознания риск умереть резко возрастает. Токи более 80–100 мА переменного и 300 мА постоянного напряжения вызывают фибрилляцию сердца и (или) прекращение работы легких. При этом наступает клиническая смерть, продолжающаяся 5–7 минут.

Величина электрического тока более 100 миллиАмпер считается смертельно опасной. Такой ток вызывает остановку дыхания и фибрилляцию сердца.

Если в течение этого времени удается оказать человеку первую помощь, он может выжить. Биологическая смерть начинается с отмирания клеток головного мозга, после чего человека уже невозможно вернуть к жизни.

Длительность протекания тока

Чем быстрее освобождают человека от действия электричества, тем больший ток он может выдержать. В приведенной ниже таблице видно, как продолжительность воздействия влияет на максимально допустимый переменный ток.

При малых токах порядка 1,1 мА частотой 50 Гц и 6 мА постоянного значения человек начинает чувствовать прохождение электричества.

В случае с переменным напряжением это будет сопровождаться слабым зудом и пощипыванием, а постоянный ток дает ощущение нагрева в месте соприкосновения с источником тока.

Если переменный ток до 5 А вызывает фибрилляцию – хаотичное сокращение сердечных мышц, то свыше 5 А сразу происходит остановка сердца. Но даже и в этом случае можно спасти человека, если действие тока было продолжительностью не более 1–2 секунды.

Почему переменный ток опаснее постоянного

Самым опасным является ток частотой 20-1 000 Гц. Он примерно в три раза опаснее постоянного напряжения. Однако при дальнейшем повышении частоты опасность переменного напряжения снижается.

Если частота превышает 500 кГц, они уже не являются смертельными, но это не значит, что человек совсем не может от них пострадать. Термическое поражение остается как от прохождения тока, так и от электрической дуги.

Остается подвести итог. На последствия от поражения электрическим током влияют: напряжение, его род, сила тока, частота переменного напряжения и сопротивление человека.

Особенно важны: в каком состоянии находится человек, его особенности, как проходит ток, и сколько времени он оказывает воздействие. Не стоит забывать и об окружающей среде, влажность и повышенная температура способствуют поражению.

Похожие материалы на сайте:

Понравилась статья — поделись с друзьями!

 

«Какая сила тока является безопасной для жизни человека?» – Яндекс.Кью

Всё условно. Что считать безопасностью? Где получено поражение? Какова влажность воздуха? От сети переменного или постоянного тока? Промышленная (50 Гц) или нет? Какова продолжительность воздействия? Всё важно Вот, например, полезный материал (https://pomegerim.ru/electrobezopasnost/dejstvie-elektricheskogo-toka-na-organizm-cheloveka.php): В быту и на производстве мы сталкиваемся с различными электроприборами, электроустановками. Соблюдая правила электробезопасности и обладая знаниями в данной сфере можно уменьшить вероятность попадания под опасное воздействие электрического тока и напряжения. В данном вопросе объединяются знания инженерного и медицинского характера, применение которых в комплексе, увеличит результат по снижению уровня электротравм дома и на производстве. Действие электрического тока на организм человека Ток, в отличие от других опасных сред, не обладает цветом, запахом, невидим. Электрический ток оказывает следующие виды воздействия на организм человека: термическое, электролитическое, биологическое. Рассмотрим каждое из этих воздействий более подробно. Термическое воздействие заключается в ожогах участков тела, нагреве сосудов и нервных окончаний. Этот вид действия называют еще тепловым. Потому что тепловая энергия, полученная из электрической образует ожоги. Электролитическое воздействие приводит к разложению крови и других жидкостей в организме посредством процесса электролиза, что вызывает нарушения в физико-химическом составе этих жидкостей. Суть повреждений сводится к молекулярному уровню – загустевание крови, изменение заряда белков, паро- и газообразование в организме. Биологическое воздействие электротока на организм сопровождается раздражением и возбуждением органов. Это вызывает судороги, сокращения. В случае с сердцем и легкими это воздействие может привести к летальному исходу по причине прекращения деятельности органов дыхания и сердца. Биологическое воздействие вызывает механические повреждения органов, суставов человека. Также механические повреждения может вызвать падение человека с высоты из-за воздействия электрического тока. Опасная, безопасная и смертельная сила тока для человека Нельзя считать какую-либо величину тока безопасной для человека. Существует лишь более и менее опасная величина электротока. Каждый человек имеет внутреннее сопротивление, на величину которого влияет множество факторов (толщина кожи, влажность помещения и тела человека, путь протекания тока). Самым опасным путем протекания тока является направление нога-голова, рука-голова, так как при этом путь идет через сердце, мозг, органы дыхания. А большая величина тока может вызвать остановку сердца и остановку дыхания. Именно эти причины являются наиболее вероятными причинами летальных исходов при протекании электротока. Считается, что постоянный ток более безопасный, чем переменный в сетях до 500В. При напряжении выше 500 вольт опасность постоянного тока возрастает. Частота сети влияет на степень тяжести электротравмы. Промышленная частота в 50 Гц является более опасной, чем частота в 500Гц. При высокой частоте наблюдается так называемый «скин-эффект», когда ток проходит не по всему проводнику, а лишь по его поверхности. А значит, внутренние органы напрямую не затрагиваются. Также на степень опасности воздействия тока на человека влияет продолжительность нахождения человека под воздействием тока. Здесь зависимость линейная – чем дольше, тем больше разрушений и неблагоприятных последствий. (См. таблицу по ссылке выше) Проходя через человеческое тело, ток может создавать электрические травмы или электрические удары. Электрический удар подразумевает, что ток возбуждает ткани организма, что вызывает их сокращение и судороги. Существует 4 группы электроударов: судороги, судороги с потерей сознания, потеря сознания с нарушением дыхания и работы сердца, клиническая смерть. При электрической травме ток наносит прямые повреждения тканям и органам человека. Это могут быть электрические ожоги, металлизация кожи, электрические метки и механические повреждения. Электрические ожоги бывают токовыми и дуговыми. Действие токового ожога связано с прохождением тока через тело человека. Дуговой ожог возникает между человеком и проводником электротока высокого напряжения, вследствие возникновения дуги между ними. Температура дуги может достигать тысяч градусов по Цельсию. Такой ожог гораздо опаснее и может плюс ко всему сопровождаться возгоранием одежды пострадавшего. Металлизация кожи происходит, когда под действием тока в кожу попадают частицы металла, при этом проводимость кожи увеличивается, что повышает травмоопасность. Электрические метки – это места, через которые ток входит и выходит из тела человека. Наиболее часто встречаются на ногах и руках. В любом случае следует стараться избегать касания токоведущих частей проводящими предметами (ловить рыбу под ЛЭП, нести стремянку вблизи шин напряжения), не использовать провода и кабели с ослабленной изоляцией, соблюдать правила безопасности при нахождении и работе в электроустановках. Берегите здоровье себя и своих родных.

ИНСТРУКЦИЯ для всех работников по электробезопасности

  1. Общие требования безопасности.

1.1  Электрический ток, проходя через тело человека, может поразить отдельные участки тела в виде ожогов и металлизации кожи или воздействовать на нервную систему и мышцы, в результате чего могут произойти судороги мышц, остановка дыхания, фибриляция ( беспорядочное подёргивание сердечной мышцы ) и остановка сердца, что в свою очередь, может привести к смертельному исходу.

1.2  Влияние электрического тока на различных людей зависит от целого ряда условий. Так, сопротивляемость человеческого тела значительно понижается, когда он работает в условиях повышенной влажности и высоких температур ( свыше +30 С ), когда человек потный, когда кожа и одежда загрязнены металлической пылью или увлажнены, когда человек утомлён, расстроен, раздражён, находится в нетрезвом состоянии .Особенно опасно попадание под напряжение, людей страдающих нервными и сердечными болезнями, так как они имеют чрезвычайно пониженную сопротивляемость электрическому току .

1.3  Люди уравновешенные, со здоровым сердцем и нервной системой, сухим, чистым телом, а также в трезвом состоянии имеют большую сопротивляемость току .

1.4  Сопротивление сухой неповреждённой кожи человека может быть до 80 000 Ом, сопротивление внутренних органов составляет 800 — 1000 Ом, поэтому расчетное сопротивление человека электрическому току принимается равным 1000 Ом. ( 1 кОм ).

1.5  Безопасным для организма человека можно считать переменный ток силой не выше 0,05 А ток силой более 0,05 — 0,1 А опасен и может вызвать смертельный исход .

1.6  Безопасным напряжением для человека считается напряжение 42 В в нормальных условиях и 12 В в условиях повышенной опасностью ( сырость, высокая температура, металлические полы и др. ).

1.7  Производственные помещения по наличию в них условий для поражения людей электротоком подразделяются на три категории: особо опасные, с повышенной опасностью и без повышенной опасности . Помещения особо опасные характеризуются наличием одновременно двух или более признаков: высокой влажностью, высокой температурой ( более 30 С ), токопроводящей пыли, токопроводящих полов, стен и др. Помещения с повышенной опасностью характеризуются одним из вышеперечисленных признаков . В помещениях без повышенной опасности указанные признаки отсутствуют.

1.8  Поражение человека электрическим током возможно в следующих случаях:

а) когда человек прикоснулся к конструкциям, находящимся под напряжением, или к одному проводнику электрического тока, а сам стоит на земле или токопроводящей конструкции;.

б) когда человек прикоснулся руками или другими частями тела одновременно к двум проводникам электрического тока, независимо от того стоит ли он на токопроводящей конструкции. Прикосновение к токопроводящим частям, находящихся под напряжением, вызывает судорожное сокращение мышц, в следствии этого пальцы пострадавшего, держащего провод руками могут так сильно сжиматься, что высвободить провод из его рук становится невозможным .

1.9  Всё электрическое оборудование и электрические приёмники, металлические корпуса рубильников и распределительных пунктов, ящиков должны иметь надёжное защитное заземление .

1.10                     Токоведущие части электрического оборудования, рубильников, распределительных щитов должны иметь надёжные кожуха, двери, не имеющие открытых отверстий, щелей и закрывающиеся на запорное устройство .

1.11                     Электропроводка должна выполнятся изолированными проводами и подвешиваться на высоте не менее 2,5 метров, если рабочее напряжение в проводе более 42 В.

1.12                     Всем работникам КАТЕГОРИЧЕСКИ ЗАПРЕЩАЕТСЯ заменять перегоревшие электролампы, плавкие вставки и другие элементы электропроводки и электрооборудования, а так же самостоятельно пытаться устранить неисправность электроприёмников. Данные виды работ производит только электромонтёр.

1.13                     Все работники автохозяйства, работающие с электроинструментом или электрооборудованием, обязаны пройти обучение и сдать экзамены на соответствующую группу допуска по электробезопасности, соответствующей их специальности.

 

2        Требования безопасности перед началом работы.

 

2.1  Для предотвращения случаев попадания работников под напряжение и поражения их электрическим током, необходимо выполнять следующие мероприятия:

2.2   Обращать внимание на предупредительные знаки и надписи по электробезопасности.

2.3  Самовольное снятие предупредительных знаков, плакатов, а также включение электроустановок при их наличии — ЗАПРЕЩЕНО!

2.4  Если перед выполнением работ необходимо включать рубильники или другие включающие пункты ( в помещениях с повышенной опасностью или особо опасных, а также в помещениях с влажной средой ), то работающие должны быть снабжены средствами индивидуальной защиты:

а) диэлектрические перчатки

б) диэлектрические коврики

в) диэлектрические калоши ( боты ) .

Эти средства должны быть проверены и иметь клеймо, в котором указана дата, до какого срока разрешено их использование и на какое напряжение .

2.5 Перед началом работы ручным электроинструментом, необходимо проверить его на наличие трещин в корпусе. Кабель для подключения ручного электроинструмента в сеть, не должен иметь заломов и задиров изоляции, вилка не должна иметь сколов. Разрешается работать только при соблюдении этих требований.

2.6  Если корпус электроинструмента металлический, работник должен быть снабжён диэлектрическими перчатками. При работе с электроинструментом с двойной изоляцией ( пластмассовый корпус ) диэлектрические перчатки не требуются .

2.7  Дпя переносных светильников в условиях ремонтных работ допускается применять напряжение только 12 В или 36 В . Лампы переносных светильников должны быть снабжены защитной сеткой . Использовать для местного освещения при ремонтных  работах напряжение 110 В или 220 В — ЗАПРЕЩАЕТСЯ !

2.8  Выдача электроинструмента и переносных светильников производится мастером или винструментальной, с обязательным фиксированием в специальном журнале, После работы инструмент возвращается с указанием возможной неисправности, если таковая имеется.

 

3        Требования безопасности во время работы.

 

3.1  При малейших ощущениях электрического тока на корпусе электрооборудования и электроинструмента необходимо сразу же отключить его и поставить в известность мастера (начальника подразделения ), вызвать электромонтёра. Приступать к работе на данном электрооборудовании не удостоверившись у мастера в том, что неисправность устранена — ЗАПРЕЩАЕТСЯ !

3.2  Во время работы не рекомендуется без необходимости прикасаться к понижающим трансформаторам, распределительным щитам, корпусам рубильников. К оголённым проводам, не имеющим изоляции прикасаться ЗАПРЕЩЕНО!

3.3  О всех замеченных неполадках в электропроводке или электрооборудовании (обрывы, оголённые провода, искрящие контакты, возгорания, запах горения электропроводки и т.д.) каждый работник должен немедленно доложить своему непосредственному руководителю.

3.4  Работники, занятые работой вблизи мест электропрогрева железобетонных конструкций прогревными трансформаторами, не должны заходить на прогреваемые места, не подлезать под ограждения и не ломать их.

3.5  Производство строительных, погрузочно — разгрузочных работ вблизи линий электропередачи и в охранной зоне ЛЭП без специального разрешения (наряд — допуска )- ЗАПРЕЩАЕТСЯ !

3.6  Все виды работ в этом случае необходимо выполнять согласно инструкции «По безопасной эксплуатации механизмов и транспорта вблизи и в охранной зоне ЛЭП и коммуникаций трубопроводов ».

3.7  В случае попадания транспорта в зону обрыва провода на земле в радиусе 5 — 10 метров или наезда автотракторной техники на опору с высоковольтными проводами, их последующего обрыва и попадания провода на корпус машины, необходимо: выходя из кабины техники, прижать руки к телу и мелкими шагами приблизится к краю кабины. Затем, выпрыгнуть из кабины, прижимая руки к телу, а ступни ног держать вместе.

Затем, очень мелкими шагами отойти на 10 — 15 метров от места обрыва провода, чтобы избежать попадания под «шаговое» напряжение . После этого доложить о случившемся диспетчеру предприятия, ответственного за высоковольтную линию, ответственному за производство работ, диспетчеру автохозяйства .

Допускается перемешаться от автомобиля лёжа, перекатываясь, прижимая руки к телу, а ноги держа вместе.

Проезд под высоковольтными линиями электропередачи машин и механизмов, имеющих общую высоту с грузом или без груза от поверхности дороги более 4,5 метров- ЗАПРЕЩАЕТСЯ !

3.8  При использовании нагревательного прибора с открытыми спиралями (элементами ) в производственных помещениях, необходимо удостоверится в его работоспособности и безопасной эксплуатации, Нагревательный прибор должен находиться не менее чем в 2 метрах от сгораемых предметов и установлен на огнестойкой подставке . Корпус нагревательного прибора должен быть надёжно заземлён . Использование нагревательных приборов с открытыми элементами в пожаро и взрывоопасных помещениях — ЗАПРЕЩЕНО!

 

4        Требования безопасности в аварийных ситуациях.

4,1 Работник должен знать порядок действий при несчастном случае и уметь оказать первую медицинскую помощь .

Последовательность действий при поражении электрическим током

а) устранить воздействие на организм поражающих факторов, угрожающих здоровью и жизни пострадавшего ( освободить от действия электрического тока, вынести из заражённой зоны, погасить горящую одежду, извлечь из воды и т.д. ), оценить состояние пострадавшего;

б) определить характер и тяжесть травмы, наибольшую угрозу для жизни пострадавшего и последовательность мероприятий по его спасению;

в) выполнять необходимые мероприятия по спасению пострадавшего в порядке срочности (восстановить проходимость дыхательных путей, провести искусственное дыхание, наружный массаж сердца, остановить кровотечение „наложить на место перелома шину, повязку и т.п.)

г) вызвать скорую медицинскую помощь (по телефону 03), врача, либо принять меры к транспортировке пострадавшего в ближайшее лечебное учреждение;

д) поддерживать основные жизненные функции пострадавшего до прибытия медицинского работника;

4.2 Первая медицинская помощь пострадавшим от электрического тока:

4.2.1. При поражении электрическим током напряжением до 1 кВ, необходимо как можно скорее освободить пострадавшего от действия тока, так как от продолжительности этого действия зависит тяжесть травмы .

Если пострадавший держит провод руками его пальцы так сильно сжимаются, что

высвободить провод из его рук становится невозможным . Поэтому нужно немедленно отключить электроустановку которой касается пострадавший . Отключение производится с помощью выключателя, рубильника или другого аппарата .

Если отключение электроустановки не может быть произведено достаточно быстро, то необходимо принять меры по освобождению пострадавшего от действия электрического тока другими способами .

Для этой цели можно использовать сухие не металлические предметы: пеньковый канат, палку, не промасленную спецодежду или перерубить провод топором, лопатой с сухой деревянной ручкой и отбросить его от пострадавшего .

При отталкивании пострадавшего нужно прежде всего изолировать руки . Лучше всего надеть диэлектрические перчатки, но можно обмотать руки прорезиненной тканью, плащом, шарфом, фуражкой или сухой спецодеждой, можно также браться за одежду пострадавшего (за полы, воротник), если она сухая и отстаёт от тела .Можно также изолировать себя встав на сухую доску или другую, не проводящую электрический ток, подстилку ( резину, свёрток одежды и т.п.).

При отделении пострадавшего от токоведущих частей рекомендуется действовать по возможности правой рукой .

4.2.2. Для освобождения пострадавшего от действия электрического тока напряжением выше 1 кВ, находящегося на земле или касающегося токоведущих частей, следует пользоваться только диэлектрическими перчатками, ботами, ковриками, специальными штангами, клещами или инструментами рассчитанными на напряжение данной установки. Когда освобождение пострадавшего от действия электрического тока вышеуказанными способами выполнить достаточно быстро и безопасно невозможно, необходимо прибегнуть к короткому замыканию и заземлению всех видов проводов линии или одного провода, которого касается пострадавший.

Следует помнить, что после отключения линии на ней может сохраниться остаточное напряжение (заряд) опасное для жизни, и что обезопасить линию может только её надёжное заземление.

4.3 Способы восстановления нормальной жизнедеятельности организма пострадавшего от воздействия электрического тока:

4.3.1. Искусственное дыхание.

Проводится в тех случаях, когда пострадавший не дышит или дышит очень плохо

( редко, судорожно, со всхлипыванием).

Наиболее эффективным считают способ «изо рта в рот» или «изо рта в нос» -. Эти способы относятся к способам искусственного дыхания по методу вдувания, при котором воздух выдыхаемый оказывающим помощь насильно подаётся в дыхательные пути пострадавшего.

Вдувание воздуха можно производить через марлю, платок, специальное

приспособление «воздуховод».

В первую очередь обеспечивают проходимость верхних дыхательных путей . Для этого гортань человека освобождают от запавшего языка или какого — либо инородного тела ( протез, песок , скопление слюны и т.д.) .После этого оказывающий помощь располагается сбоку от пострадавшего, одну руку подсовывает под шею пострадавшего, а ладонью другой руки надавливает на его лоб, максимально запрокидывает голову .При этом корень языка поднимается и освобождается гортань, а рот пострадавшего открывается .Затем оказывающий

помощь делает глубокий вдох, полностью охватывает губами открытый рот пострадавшего и делает энергичный выдох, с некоторым усилием вдувая воздух в его рот, одновременно закрывая его нос щекой или пальцами руки, находящейся на лбу. Как только грудная клетка поднялась, нагнетание воздуха приостанавливают, происходит пассивный выдох у пострадавшего .

Данную операцию производят до получения положительного результата (покраснения кожи, а так же выход больного из бессознательного состояния и появления у него самостоятельного дыхания).

Интервал между искусственными вдохами должен составлять 5 секунд (12 дыхательных циклов в минуту . Если челюсти пострадавшего плотно стиснуты, необходимо прибегнуть к способу «изо рта в нос», который производится идентично вышеописанному способу. Эффективным способом оживления пострадавшего является чередование искусственного дыхания и наружного массажа сердца.

4,3.2. Наружный массаж сердца.

При поражении человека электрическим током может наступить не только остановка дыхания, но и прекратиться кровообращение, когда сердце не обеспечивает циркуляции крови в организме .Поэтому необходимо возобновить кровообращение искусственным путём .

При остановке сердца, не теряя ни минуты, пострадавшего нужно уложить на ровное жёсткое основание: скамью, пол, в крайнем случае положить под спину доску ( никаких валиков под плечи и шею подкладывать нельзя ) .

Если помощь оказывает один человек, он располагается сбоку от пострадавшего и, наклонившись, делает два быстрых энергичных вдувания ( по способу «изо рта в рот» или «изо рта в нос»), затем приподнимается, оставаясь на этой же стороне от пострадавшего, ладонь одной руки кладет на нижнюю половину грудины (отступив на два пальца от её нижнего края), а пальцы поднимает. Ладонь второй руки он кладёт поверх первой поперёк или вдоль и накладывает, помогая натиском своего корпуса. Руки при надавливании должны быть выпрямлены в суставах локтей. Надавливание следует производить толчками, чтобы смещать грудину на 4-5 см, продолжительность надавливания не более 0,5 с, интервал между отдельными надавливаниями 0,5 с. В паузах рук с грудины не снимают, пальцы остаются прямыми, руки полностью выпрямлены в локтевых суставах .

На каждые 2 вдувания производится 15 надавливаний на грудину. За одну минуту

необходимо сделать не менее 60 надавливаний и 12 вдуваний воздуха.

4.4. Помощь пострадавшим при электрических ожогах .

При оказании помощи пострадавшему, во избежании заражении нельзя касаться

руками обожженных участков кожи или смазать их мазями, жирами, маслами, вазелином присыпать питьевой содой и т. д. Нельзя вскрывать пузыри, приставшую к обожжённому месту мастику, канифоль или другие смолистые вещества, т. к. можно содрать обожженную кожу и получить заражение раны.

При небольших по площади ожогах 1 и 2 степеней нужно положить на обожженный участок кожи стерильную повязку. Одежду и обувь с обожженного места нельзя срывать, а необходимо разрезать ножницами. Если куски одежды прилипли к обожженной коже, то поверх них следует наложить стерильную повязку и направить пострадавшего в лечебное учреждение .

При тяжёлых и обширных ожогах пострадавшего необходимо завернуть в чистую

простыню или ткань, не раздевая его, укрыть потеплее, напоить тёплым чаем и

обеспечить покой до прибытия врача.

Обожженное лицо необходимо закрыть стерильной марлей .

При ожогах глаз следует делать холодные примочки из раствора борной кислоты

( половина чайной ложки кислоты на стакан воды) и немедленно вызвать скорую

помощь.

4.5. Оказание первой медицинской помощи при отравлении угарными газами в следствии возгорания изоляции электропровода и кабелей .

4.5.1. При отравлении угарными газами, возникающими по причине горения изоляции кабеля или обмотки трансформатора, а так же двигателя, необходимо пострадавшего положить на спину, расстегнуть воротник . Обеспечить свободный доступ свежего воздуха . Пострадавшего следует укрыть теплее и давать нюхать нашатырный спирт. У пострадавшего в бессознательном состоянии может возникнуть рвота, поэтому необходимо повернуть его голову в сторону. Вызвать скорую помощь по телефону 03 .

При возможной остановке дыхания следует сразу же начать делать искусственное

дыхание.

 

5        Требования безопасности по окончании работы

5.1  Отключить все электроаппаратуры, электрооборудование, электроинструмент и другие переносные электроприёмники .

5.2  Сдать электроинструмент на склад или в инструментальную.

5.3  Доложить об окончании работ мастеру или бригадиру.

5.4  Убрать рабочее место.

5.5  После уборки вымыть лицо и руки с мылом.

 

вернуться

опасная для жизни величина ампер и вольт

По мнению опытных электриков, электроток опасен тем, что он невидим. Электричество, воздействующее на человеческий организм, вызывает тяжелые последствия, вплоть до смертельного исхода. Установили, что ток 50-100 мА опасен для жизни, а более 100 мА – смертелен. Речь идет о токах, проходящих через человека. В этой статье разберемся, почему переменный ток опаснее постоянного.

Знак высокого напряжения

Исход поражения электротоком

Ситуации бывают различными, поэтому исход от удара током наблюдается разнообразный. При получении сильного электрического удара вызываются проблемы с кровообращением и дыханием. Тяжелые случаи характеризуются сердечной фибрилляцией: мышцы сердца хаотично подергиваются. Фактически сердце перестает нормально функционировать, поэтому в такой ситуации требуется скорейшее медицинское вмешательство.

Зачастую поражение электротоком имеет силу до 1000 В. Ожоги возникают, если сила превышает 1 А. Наиболее частая причина – несоблюдение человеком правил техники безопасности. Элемент, по которому проходит электричество, находится вблизи человеческого тела, в результате чего возникает искровой разряд, приводящий к ожогам различной степени. При случайном получении искрового разряда ток, контактирующий с телом, нагревает ткань до 60 градусов Цельсия. Начинает сворачиваться белок, а впоследствии на пораженном участке появляется ожог. Электрические ожоги опасны, так как вылечить их довольно проблематично.

Удар электротоком может иметь различные последствия

Опасные величины тока

Поражение электричеством бывает разным, на что влияет три фактора:

  • Какова частота: постоянный или переменный;
  • Сила;
  • В каком направлении движется, проходя через тело.

Электроток делят также, в зависимости от того, как он влияет на человеческое здоровье:

  • Ощутимый – только раздражает кожу. Безопасная величина – не более 0.6 милиампер;
  • Неотпускающий – переменный с периодическими импульсами, из-за которых человек «прилипает» к источнику электричества. Случается, если сила тока превышает 0.025 ампер;
  • Фибрилляционный – из-за него вызывается фибрилляция внутренних органов, в первую очередь, сердца. Если сила электричества превышает 0.1 ампер, орган может остановиться.

Необходимо знать! Человеческий организм сопротивляется электричеству. Сила удара зависит от многих факторов: состояние здоровья потерпевшего во время удара, психическое состояние и даже качество обуви. Отталкиваясь от величин электрического сопротивления, выводят показания напряжения тока, опасные для человека.

Отталкиваясь от техники безопасности, опасные следующие показатели напряжения:

  • 65 вольт – жилые помещения и общественные здания, которые отапливаются и имеют внутреннюю влажность до 60%;
  • 36 вольт – помещения с повышенным уровнем влажности (до 75%). Это подвальные помещения, кухни и так далее;
  • 12 вольт – очень влажные пространства (100%): бассейн, баня, прачечная, котельная и так далее.

Обратите внимание! Частота электротока также играет роль. Опасным для человека считается значение от 50 до 60 герц.

Опасность переменного и постоянного тока

Известно, что электроток бывает постоянный и переменный, но не каждый житель понимает между ними разницу и знает, какой оказывает более серьезное воздействие на организм. На вопрос, какой ток опаснее, специалисты отвечают – переменный.

Прохождение по телу

Объясняется это тем, что постоянный электроток должен быть в три раза мощнее переменного, чтобы быть смертельно опасным для человеческого здоровья. Переменный – более быстрый и сильный, что больше сказывается на нервных окончаниях и мышечной ткани (в первую очередь, сердечной). Электрическое сопротивление людей покрывает мощность постоянного тока (силой не выше 50 милиампер). В случае с переменным электротоком граница опускается до 10 милиампер. Если электрическое напряжение достигает 500 вольт, то оба вида тока оказывают одинаковый вред. Если показатель повышается, более опасный в такой ситуации постоянный электроток.

Биологическое действие электричества напрямую зависит от того, с какой интенсивностью организм ему подвергается, а это важный фактор, из-за которого возникает фибрилляция желудочков сердца. Смертельный электрический ток для человека – длительное прикосновение к электропроводникам с силой 0.25-80 мА. При этом вызываются судороги дыхательных мышц и как следствие – острая асфиксия.

Электричество распространяется по организму лишь в том случае, если есть точка входа и выхода тока. То есть одновременно нужно прикоснуться к двум электродам. Речь идет о двуполюсном включении или соприкосновении с одним электродом. Если часть тела человека заземлена, то такое включение называют однополюсным. Бывает и частичное включение, при котором изолированный от земли человек прикасается к разноименным полюсам. В таком случае он пройдет через включенный отрезок руки, а это, как правило, не опасный ток. Если имеет место высокое напряжение, то электротоком может поразить, даже если нет прямого контакта с проводником: то есть на расстоянии, посредством дугового контакта, который возникает, если к нему приблизиться. Ионизация воздуха является причиной того, что человек контактирует с установками или проводами, по которым проходит электроэнергия. Ток электричества опасный для человека особенно в сырую погоду, так как электропроводимость воздуха повышена. В случае со сверхвысоким напряжением величина электрической дуги достигает длины в 35 см.

Электрический ток опасен для человеческого организма, поэтому нужно соблюдать элементарные требования техники безопасности. Сам он бывает постоянным и переменным, каждый по-своему воздействует на человека. Безопасная работа с электроустановками – соблюдение всех правил и использование средств защиты.

Видео

Какой электрический ток опаснее для человека: переменный или постоянный

Сегодня мы расскажем читателям сайта Сам Электрик, какой электрический ток опаснее для человека: переменный или постоянный. Сразу же следует отметить, что на этот вопрос можно дать ответ только в индивидуальном порядке, если известны сила, частота, и что не менее важно – напряжение. Далее мы предоставим к Вашему вниманию основные значения, при которых опасность переменного и постоянного электрического тока будет наименьшей и, соответственно, небезопасной для жизни.

Разница между двумя сравниваемыми вариантами

Итак, первым делом вкратце объясним Вам, в чем отличие переменного тока от постоянного. Основное отличие заключается в том, что в первом случае направленное движение заряженных частиц будет проходить прямо, а во втором – в хаотичном направлении. На графике ниже наглядно показана разница между двумя сравниваемыми вариантами и в то же время предоставлено краткое описание о том, как протекает переменный и постоянный электроток в цепи.

Помимо этого следует добавить, что постоянный ток в домашних условиях чаще всего протекает в светильниках – к примеру, если в комнате предусмотрена скрытая подсветка светодиодной лентой. В то же время переменный протекает во всех розетках, распределительная коробках и щитке, поэтому его опасность для жизни человека более актуальна.

Опасные для человека значения

Как мы сказали ранее, опасность электротока для жизни человека зависит от того, какое значение напряжения и частоты колебания будет протекать в цепи. Чтобы корректно ответить на вопрос, какой ток более опасен, рассмотрим все возможные значения и их диапазоны.

  1. Частота колебаний. В бытовой электрической сети составляет 50 Гц. При частоте от 10 до 500 Гц переменный ток одинаково опасен для человека. В диапазоне от 500 до 1000 Гц опасность заметно возрастает. Переменный электрический ток с частотой колебаний свыше 1000 Гц менее опасен для жизни. Тут же следует отметить, что постоянный эл.ток примерно в 3-4 раза безопаснее переменного, если частота колебаний последнего составляет 50 Гц.
  2. Напряжение. Если напряжение в сети не превышает 400 Вольт, то в этом случае переменный электрический ток опаснее постоянного. В диапазоне 400-600 Вольт сравниваемые варианты примерно одинаково опасны для жизни человека. Если напряжение в сети на порядок выше 500 Вольт опасность постоянного электротока возрастает и в этом случае переменный считается не таким опасным.

Также следует отдельно обратить Ваше внимание на такую величину, как силу тока. Этот параметр считается безопасным, если при переменном токе не превышает 10 мА, а при постоянном 50 мА. Если сравнивать опасность по Амперам, то тут можно с уверенностью сказать, что при одинаковых значениях переменный будет опаснее для человека, нежели постоянный.

Вот и все, что хотелось рассказать Вам по поводу данного вопроса. Надеемся, что Вы осознаете всю опасность воздействия электричества и при электромонтажных работах максимально серьезно подходите к обеспечению электробезопасности! Так или иначе, для бытовых условий можно с уверенностью ответить на вопрос, какой электрический ток опаснее для человека. Если постоянный ток используется только в освещении, то он не такой опасный, как переменный (в розетках, распределительных коробках и щитке)! Рекомендуем также ознакомиться с не менее важной статьей – какие инструменты должны быть у домашнего электрика!

Сколько вольт или ампер может убить человека?

Человека убивает не напряжение, а ток. Люди умирали при низком напряжении 42 вольт. Время также является фактором. Ток в 0,1 ампера всего за 2 секунды может быть смертельным. Поскольку напряжение = ток x сопротивление, ток зависит от сопротивления тела. Внутреннее сопротивление между ушами составляет всего 100 Ом, в то время как при измерении от пальца до ног оно составляет около 500 Ом.

В физических комедиях часто изображают поражение электрическим током, и сюжет развивается обычным образом: главный герой комикса случайно попадает в провод, не зная, какой сильный ток течет по нему.Он получает смертельный шок, который приводит к стереотипному шимми, обугленному лицу и волосам, которые заканчиваются, как зонтик, повернутый внутрь ветром.

Вопрос, почему этот несчастный случай со смертельным исходом воспринимается как юмористический, тревожит… интересно, но тревожит. Правдоподобный ответ можно найти здесь. Однако на данный момент этот дискурс неуместен. Что нас беспокоит, так это то, почему мы совсем не нечувствительны к электричеству и сколько его на самом деле убьет нас.

Почему высокое напряжение считается опасным?

Это, конечно, важные знания в целях безопасности.На электрических платах и ​​генераторах мы находим предупреждающие сообщения с общим символом опасности: человеческий череп, парящий над двумя скрещенными костями.

Этот символ сопровождается рейтингом этого устройства, который указывает на высокое напряжение, под которым он работает, и дает вам знать, что вы, вероятно, погибнете при контакте с ним. Использование напряжения заложило в нас психологическую тенденцию.

Теперь мы считаем, что 10 000 вольт будут более смертоносными, чем 100 вольт. Однако это верно лишь отчасти.

Поражение электрическим током часто может происходить при домашнем напряжении 110 вольт, а в некоторых случаях даже при 42 вольт!

Конечно, большее напряжение потребляет больше тока, но нас убивает не калибр, а пуля, которую она стреляет. Каким бы ни было напряжение, истинная причина смерти — это ток, проталкиваемый через тело.

По этой же причине птицы, отдыхающие на проводах, не получают удар током. (Кредиты: palickam / Shutterstock)

Однако мы не должны полностью отказываться от напряжения, потому что без напряжения или разности потенциалов вообще не было бы тока.Следовательно, повешение на проводе не приведет к поражению электрическим током , если не коснется земли. Свешивание с проводом создает уравнивание потенциала с проводом, тогда как прикосновение к земле немедленно создает разность потенциалов, которая пропускает через жертву огромный ток.

Итак, сколько электричества нас убьет?

Поражение электрическим током: сколько электричества убьет вас?

Ток 10 мА или 0,01 А — это серьезное поражение, но не со смертельным исходом. По мере приближения к 100 мА или 0.1 А, начинаются сокращения мышц. Необходимо понимать, что из-за низкого сопротивления сердца тока всего 10 мА достаточно, чтобы нас убить.

Но ток никогда не достигает сердца, так как сопротивление нашей кожи выше и, таким образом, полностью поглощает этот ток. Если этот скудный поток каким-либо образом достигнет сердца, это почти наверняка будет фатальным.

Когда ток превышает 1000 мА или 1 А, сокращения мышц усиливаются до такой степени, что мы не можем освободить провод.Эта упорство, по иронии судьбы, является следствием мышечного паралича.

В этот момент сердце испытывает фибрилляцию желудочков, некоординированное, прерывистое подергивание желудочков, которое вызывает неэффективное сердцебиение, которое может привести к смерти, если не будет вызвана немедленная помощь.

Дальнейшее увеличение тока до 2000 мА или 2 А приводит к ожогам и потере сознания. Сокращение мышц, вызванное потрясением, теперь настолько сильное, что сердце сжимается.Воздействие такого количества тока может привести к ужасным внутренним ожогам, а зажимы — к остановке сердца. Смерть возможна.

Зажимной механизм, однако, удивительно прибылен, поскольку защищает сердце от фибрилляции желудочков. Шансы на выживание невелики, но их можно компенсировать немедленной медицинской помощью пострадавшему. Дефибрилляторы — это медицинские устройства, которые врачи используют для спасения жертв шока.

Эффекты можно резюмировать в табличной форме следующим образом:

Почему мы нечувствительны к току?

Хотя для протекания тока требуется определенное напряжение, количество тока, протекающего в нашем теле, зависит от того, насколько проницаемо тело для тока или просто от его сопротивления.Устойчивость к току различается в зависимости от состояния кожи — сухой или влажной. Он оценивается в 1000 Ом для влажной кожи и более 500 000 Ом для сухой кожи.

Сопротивление также зависит от точки контакта. Внутреннее сопротивление между ушами составляет всего 100 Ом, при измерении от пальца до стопы оно составляет около 500 Ом. Благодаря этому конечному сопротивлению мы нечувствительны к току.

Статьи по теме

Статьи по теме

Еще один важный фактор — время.Степень испытания зависит от того, как долго тело подвергается воздействию определенного тока. Например, ток в одну десятую ампера может быть смертельным всего за 2 секунды.

Как насчет ответа на несколько вопросов об электричестве?

Посмотрим, насколько хорошо вы разбираетесь в токах и напряжениях

Начать викторину

Ваш ответ:

Правильный ответ:

Далее

У вас {{SCORE_CORRECT}} из {{SCORE_TOTAL}}

Пройти тест еще раз

Безопасность и гигиена труда в электротехнике (Пособие для учащихся)

Тяжесть поражения электрическим током зависит от количества ударов электрическим током. ток и продолжительность времени, в течение которого ток проходит через тело.Для Например, 1/10 ампера (Ампер) электричества, проходящего через тело для всего 2 секунды достаточно, чтобы вызвать смерть. Величина внутреннего тока человек может выдерживать и при этом контролировать мышцы руки и стрелка может быть меньше 10 миллиампер (миллиампер или мА). Токи выше 10 мА может парализовать или «заморозить» мышцы. Когда это «замораживание» происходит, человек больше не может освободить инструмент, проволоку или другой предмет. Фактически, наэлектризованный объект может удерживаться еще сильнее, в результате чего при более длительном воздействии шокового тока.По этой причине ручные инструменты это может быть очень опасно. Если ты не можешь отпустить инструмент, ток продолжается через ваше тело в течение более длительного времени, что может привести к к параличу дыхания (мышцы, контролирующие дыхание, не могут двигаться). Вы перестаете дышать на какое-то время. Люди перестали дышать, когда был поражен током от напряжения до 49 вольт. Обычно требуется ток около 30 мА, чтобы вызвать паралич дыхания.

Токи более 75 мА вызывают фибрилляцию желудочков (очень быстро, неэффективное сердцебиение).Это состояние приведет к смерти в течение нескольких минут. если для спасения жертвы не используется специальное устройство, называемое дефибриллятором. Паралич сердца возникает при 4 амперах, что означает, что сердце не перекачивает все. Ткань обжигается током более 5 ампер. 2

В таблице показано, что обычно происходит для диапазона токов (длительный второй) при типичных бытовых напряжениях. Более длительное время выдержки увеличивает опасность для пострадавшего от электрошока.Например, ток 100 мА применяется для 3 секунды так же опасны, как ток 900 мА, приложенный к дробной части. секунды (0,03 секунды). Мышечная структура человека также составляет разница. Люди с меньшим количеством мышечной ткани обычно страдают при более низкой текущие уровни. Даже низкое напряжение может быть чрезвычайно опасным, потому что степень травмы зависит не только от силы тока, но и от время, в течение которого тело находится в контакте с цепью.

НИЗКИЙ НАПРЯЖЕНИЕ НЕ ОЗНАЧАЕТ НИЗКОЙ ОПАСНОСТИ!


Дефибриллятор в употреблении
  • ампер (ампер) — единица измерения силы тока.
  • миллиампер (миллиампер или мА) — 1/1000 ампера
  • шокирующий ток — электрический ток, который проходит через часть тела
  • Вы будет больнее, если вы не сможете отпустить инструмент, дающий шок.
  • В чем дольше шок, тем серьезнее травма.
  • Высокая напряжение вызывает дополнительные травмы!
  • Высшее напряжения могут вызвать большие токи и более сильные удары.
  • Некоторые травм от поражения электрическим током не видно.

  • Эффекты электрического тока * на теле 3

    Текущий Реакция
    1 миллиампер Просто обморок покалывание.
    5 миллиампер Легкий шок чувствовала. Тревожно, но не больно. Большинство людей могут «отпустить». Однако сильные непроизвольные движения могут стать причиной травм.
    6-25 миллиампер (женщины) † Болезненный шок. Мышечный контроль потерян. Это диапазон, в котором «замораживание токи ».Может быть, невозможно «отпустить».
    9-30 миллиампер (мужчины)
    50–150 миллиампер Чрезвычайно болевой шок, остановка дыхания (остановка дыхания), тяжелая мышца схватки. Мышцы-сгибатели могут вызывать удержание; мышцы-разгибатели может вызвать сильное отталкивание. Смерть возможна.
    1,000- 4300 миллиампер (1-4,3 ампера) желудочковый возникает фибрилляция (неритмичное сердцебиение). Мышцы договор; происходит повреждение нервов. Вероятна смерть.
    10 000 миллиампер (10 ампер) остановка сердца возникают сильные ожоги.Вероятна смерть.
    15 000 миллиампер (15 ампер) Самый низкий максимальный ток при котором обычный предохранитель или автоматический выключатель размыкает цепь!
    * Эффекты предназначены для напряжений менее 600 вольт. Более высокие напряжения также вызвать сильные ожоги. † Различия в содержании мышц и жира влияют на тяжесть шока.

    Иногда высокий напряжения приводят к дополнительным травмам. Высокое напряжение может вызвать сильное мышечные сокращения. Вы можете потерять равновесие и упасть, что может вызвать травму или даже смерть, если вы упадете в механизм, который может раздавить ты. Высокое напряжение также может вызвать серьезные ожоги (как показано на страницах 9 и 9). 10).

    При 600 вольт ток через тело может достигать 4 ампер, вызывая повреждение внутренних органов, таких как сердце.Высокие напряжения также производить ожоги. Кроме того, могут образовываться тромбы внутренние кровеносные сосуды. Нервы в зоне контакта могут быть повреждены. Мышечные сокращения может вызвать переломы костей либо из-за самих сокращений, либо из-за от водопадов.

    Сильный шок может нанести гораздо больший вред телу, чем это видно. Человек может страдать внутренним кровотечением и разрушением тканей, нервов, и мышцы.Иногда скрытые травмы, вызванные поражением электрическим током привести к отсроченной смерти. Шок — это часто только начало цепочки. событий. Даже если электрический ток слишком мал, чтобы вызвать травму, ваша реакция на шок может привести к падению и появлению синяков, сломанные кости или даже смерть.

    Продолжительность разряда сильно влияет на количество травм. Если шок непродолжительный, он может быть только болезненным.Более длинный шок (длящийся несколько секунд) может быть фатальным, если уровень ток достаточно высок, чтобы вызвать фибрилляцию желудочков в сердце. Это не так много тока, когда вы понимаете, что небольшая дрель использует В 30 раз больше тока, чем то, что убьет. При относительно больших токах смерть неизбежна, если шок будет достаточно продолжительным. Однако если шок короткий и сердце не повреждено, нормальное сердцебиение может возобновить, если контакт с электрическим током устранен.(Этот тип восстановления бывает редко.)

    Сумма тока прохождение через тело также влияет на тяжесть электрического шок. Чем выше напряжение, тем больше ток. Итак, есть большее опасность сверху
    напряжения. Сопротивление препятствует току. Чем ниже сопротивление (или импеданс в цепях переменного тока), тем больше будет ток. Сухая кожа может иметь сопротивление 100 000 Ом и более.Мокрый
    кожа может иметь сопротивление всего 1000 Ом. Влажные условия труда или сломанная кожа резко снизит сопротивление. Низкое сопротивление влажной кожи позволяет току легче проходить в тело и давать больший шок. Когда к точке контакта или когда площадь контакта больше, сопротивление ниже, что приводит к более сильному потрясения.

    Электродрели используйте в 30 раз больше тока, чем то, что убивает.

    Путь электрический ток через тело влияет на силу удара. Наиболее опасны токи, проходящие через сердце или нервную систему. Если вы касаетесь головой провода под напряжением, ваша нервная система будет поврежден. Прикосновение к токоведущей электрической части одной рукой — в то время как вы заземлены с другой стороны вашего тела — вызовет электрический ток проходит через вашу грудь, что может повредить ваше сердце и легкие.

  • Большее ток, тем сильнее шок!
  • Степень серьезности Степень удара зависит от напряжения, силы тока и сопротивления.
  • сопротивление- способность материала уменьшать или останавливать электрический ток
  • Ом единица измерения электрического сопротивления
  • Нижний сопротивление вызывает большие токи.
  • Токи через грудь очень опасны.

  • Мужчина сервисный техник прибыл на дом к заказчику для выполнения предзимний ремонт на масляной печи. Затем клиент ушел дом и вернулся через 90 минут.Она заметила сервис грузовик все еще стоял на подъездной дорожке. Еще через 2 часа заказчик вошел в лазарет с фонариком, чтобы найти техника но не мог его видеть. Затем она позвонила владельцу компании, кто пришел в дом. Он обыскал пространство для обхода и нашел техника на животе, опираясь на локти перед печь. Был вызван и объявлен помощник коронера округа. техник мертв на месте.У пострадавшего были электрические ожоги. на его скальпе и правом локте.

    После инцидента электрик осмотрел место происшествия. Переключатель выключатель, который предположительно регулирует электрическую мощность в печи находился в положении «выключено». Электрик описал проводка как «случайная и запутанная».

    Две недели спустя окружной электротехнический инспектор выполнил еще одну осмотр. Он обнаружил, что неправильная проводка тумблера позволял току течь в печь, даже когда переключатель был в положение «выключено».Владелец компании заявил, что потерпевший был очень скрупулезным работником. Возможно, жертва исполнила больше обслуживания печи, чем предыдущие техники, подвергая сам к электрике
    опасность.

    Эту смерть можно было предотвратить!

    • В Пострадавший должен был проверить цепь, чтобы убедиться, что она обесточена.
    • Работодатели должны обеспечить рабочих соответствующим оборудованием и обучением.Использование защитного оборудования должно быть требованием работы. В в этом случае простой тестер цепей мог спасти жертву жизнь.
    • Жилая электропроводка должна соответствовать Национальным электротехническим нормам и правилам (NEC). Несмотря на то что NEC не имеет обратной силы, все домовладельцы должны убедиться, что их системы безопасны.

    NEC N национал. E электрический C ode —
    исчерпывающий перечень методов защиты рабочих и оборудования от поражения электрическим током, например, пожара или поражения электрическим током
    Электрический ожог на руке и руке

    Были случаи сильного ожога руки или ноги электрическим током высокого напряжения. ток до точки отрыва, и пострадавшего не ударит током.В этих случаях ток проходит только через часть конечности, прежде чем он выходит из тела в другой проводник. Таким образом, нынешний не проходит через область груди и не может вызвать смерть, даже если жертва сильно изуродована. Если ток проходит через грудь, человек будет почти
    обязательно быть пораженным электрическим током. Большое количество тяжелых электротравм. включают прохождение тока от рук к ногам.Такой путь предполагает и сердце, и легкие. Этот тип шока часто заканчивается летальным исходом.

    Плечо с ожогом третьей степени от высоковольтной линии.

    Сводка Раздела 2

    Опасность поражения электрическим током зависит от •••

    количество электрического тока через тело,
    продолжительность электрического тока через тело, и
    путь электрического тока через тело.

    Что такое безопасное напряжение и безопасный ток человеческого тела? — Новости

    25 мая 2019

    Напряжение безопасности человеческого тела

    Отраслевые нормы Безопасное напряжение не выше 36 В, постоянное безопасное напряжение контакта 24 В, безопасный ток 10 мА. Степень повреждения человеческого тела, вызванного поражением электрическим током, в основном зависит от величины тока, протекающего через человеческое тело, и продолжительности включения питания. Чем больше сила тока, тем больше смертельный риск; чем больше продолжительность, тем больше вероятность смерти.Минимальное значение тока, которое можно почувствовать, называется током считывания. Переменный ток составляет 1 мА, а постоянный ток — 5 мА. Максимальный ток, от которого можно избавиться после электрошока, называется током, переменный ток — 10 мА, постоянный ток — 50 мА, а постоянный — 50 мА. Опасный для жизни ток называется смертельным током, а смертельный ток составляет 50 мА. В случае защиты от поражения электрическим током допустимый ток человеческого тела обычно составляет 30 мА.

    Какая причина поражения электрическим током?

    1, поражение электрическим током

    Смерть человека или животного вызвана прохождением электрического тока в человеческом теле, который нарушает биоэлектричество нервной проводимости человеческого тела, в результате чего мозг теряет контроль тела, или чувствовать ненормальную стимуляцию, и давать неправильные команды мышцам и органам.Особенно, когда ток проходит через сердце, сердце сядет на корточки и перестанет биться, что приведет к гибели человеческого тела из-за нехватки кислорода. Дело не в том, что люди, у которых есть ток, будут поражены электрическим током, в зависимости от силы тока. Сила тока равна напряжению, разделенному на сопротивление человеческого тела. Почему батарея не разряжена? Поскольку напряжение батареи слишком низкое, ток, генерируемый в человеческом теле, слишком мал, люди не чувствуют и не могут причинить вред людям.Чем больше ток, тем больше вероятность того, что человека ударит током. Это означает, что чем выше напряжение, тем опаснее.

    2, электрические ожоги

    Когда человеческое тело подвергается воздействию высокого напряжения, в теле генерируется большой ток, который может вызвать ожоги, ожоги и т. Д. Тела из-за теплового воздействия тока . Видно, что независимо от вида поражения электрическим током, главной причиной смерти людей и животных является прохождение электрического тока через тело.Необходимым условием для генерации тока является создание замкнутого контура.

    Принцип безопасного напряжения

    Согласно закону Ома (I = U / R) можно знать, что величина тока, протекающего через тело человека, зависит от приложенного напряжения и сопротивления тела.

    В дополнение к сопротивлению человека сопротивление человеческого тела также должно быть связано с одеждой, обувью, штанами и другим электрическим сопротивлением вне человеческого тела. Хотя сопротивление человеческого тела обычно может достигать 5000 Ом, существует множество факторов, влияющих на сопротивление тела, таких как влажная кожа и пот, с проводящей пылью, площадью контакта и давлением заряженного тела, а также влажным маслом одежды, обуви и т. Д. носки могут снизить сопротивление тела, поэтому ток, протекающий через тело человека, невозможно рассчитать заранее.

    Следовательно, чтобы определить условия безопасности, ток безопасности часто не используется, но напряжение безопасности используется для оценки: при нормальных обстоятельствах, то есть в среде, где существует риск высыхания и риск Поражение электрическим током небольшое, безопасное напряжение регулируется до 24 В, а поражение электрическим током опасно для влажности. В более крупных условиях (например, металлические контейнеры, сварка и ремонт труб) безопасное напряжение регулируется до 12 В, так что ток через тело человека во время поражения электрическим током может быть ограничен небольшим диапазоном, что может защитить личную безопасность до определенная степень..

    Когда сопротивление тела постоянно, чем выше напряжение, с которым контактирует человеческое тело, тем больше ток, проходящий через человеческое тело, и тем серьезнее повреждение человеческого тела. Но дело не в том, что люди могут причинить вред человеческому телу, прикоснувшись к источнику питания. В повседневной жизни мы касаемся руками полюсов обычных сухих батарей, а человеческое тело не испытывает никаких ощущений. Это связано с тем, что напряжение обычных сухих батарей низкое (1,5 В постоянного тока). Когда напряжение, приложенное к человеческому телу, ниже определенного значения, напряжение не вызовет серьезных повреждений человеческого тела за короткое время.Мы называем это напряжение безопасным напряжением.

    Что такое безопасное напряжение и безопасный ток человеческого тела?

    Безопасное напряжение:

    36 В в случае сухости, 24 В в небольшом количестве и только 12 В в особенно влажных условиях. 24 В опасно при нормальных условиях окружающей среды.

    Безопасный ток:

    Безопасное напряжение и ток синхронизированы. Если напряжение достигает 1000 В, а ток составляет всего 1 мА, это не причинит вреда человеческому телу.Например, в настоящее время обычно используется электронная мышеловка высокого напряжения, низкого тока. Люди сталкиваются с онемением, но большой опасности нет.

    В нормальных условиях, ток ниже 15 мА, люди могут бодрствовать, вы можете избавиться от них. Однако ток 15 мА и более опасен для человеческого организма. В зависимости от влажности окружающей среды он может вызвать затруднения при избавлении от него и затруднение дыхания.

    Безопасность электричества относительна, в основном зависит от вашего контакта и проводимости воздуха и окружающей среды.

    Откуда берется безопасное напряжение 36В для человеческого тела?

    Величина сопротивления тела в основном зависит от полноты и худобы тела и практически не зависит от внешних факторов. Его значение составляет около 500 Ом, но сопротивление кожи сильно варьируется в зависимости от условий, поэтому сопротивление тела также находится в большом диапазоне. Изменения внутри.

    На электрическое сопротивление тела влияет множество факторов. Кожа не только толстая, но и влажная, потная, поврежденная, а токопроводящая пыль снижает сопротивление тела.Увеличенная площадь контакта и повышенное контактное давление также уменьшат сопротивление тела. Повышенное контактное напряжение пробивает роговой слой эпидермиса и увеличивает электролиз тела, что также снижает сопротивление тела.

    Кроме того, сопротивление тела также уменьшается с увеличением частоты сети, например, сопротивление тела при 100 кГц составляет примерно половину от сопротивления при 50 Гц. В нормальных условиях сопротивление тела можно рассматривать в диапазоне 1000 ~ 1500 Ом, самое низкое можно измерить в 800 Ом.Согласно экспериментам и анализу, предельный ток частоты сети, допустимый для человеческого тела, составляет около 50 мА, поэтому его можно рассчитать в соответствии с законом Ома, U = IR≈0,05 & TImes; 800 = 40 (В), и максимально допустимой частотой сети. по человеческому телу известно. Напряжение примерно 40 В.

    Таким образом, безопасное напряжение в Китае регулируется на уровне 36 В и 12 В. Для мест с плохими условиями труда, таких как большие трубы, шахты, котлы и другие металлические контейнеры с хорошей электропроводностью, меньшим сопротивлением тела или увеличенными возможностями контакта, безопасное напряжение следует установить ниже, обычно это 24 В или 12 В.Для 12 В это также называется абсолютным безопасным напряжением.

    Рейтинг безопасного напряжения Китая

    Национальный стандарт «Безопасное напряжение» предусматривает, что рейтинг безопасного напряжения Китая составляет 42 В, 36 В, 24 В, 12 В и 6 В, которые следует выбирать в зависимости от рабочего места, условий оператора, использования. режим, режим питания, состояние линии и другие факторы. Безопасное напряжение — это напряжение, которое не вызывает физического удара, обычно менее 36 вольт.

    Безопасное напряжение — это напряжение, не вызывающее прямой смерти или инвалидности.«Безопасное сверхнизкое напряжение», обеспечивающее непрерывный контакт в нормальных условиях окружающей среды, составляет 24 В. (также может быть 36 В, 12 В переменного / постоянного тока, 24 В является наиболее распространенным)

    Национальный стандарт «Безопасное напряжение» (GB3805-83) предусматривает, что номинальное напряжение безопасности Китая составляет 42 В, 36 В, 24 В, 12 В и 6 В, которое следует выбирать в зависимости от рабочего места, условий работы оператора, режима использования, режима питания, состояния линии и других факторов. .

    24 В в зданиях с высоким риском поражения электрическим током.

    Это 12 В в зданиях с особой опасностью поражения электрическим током.

    Смертельный удар электрическим током: какое напряжение вызывает смерть?

    Вопрос с подвохом. Само по себе напряжение — не единственный фактор, способствующий серьезности поражения электрическим током. Ток, обычно измеряемый в амперах, также является важной частью уравнения, наряду с другими второстепенными факторами.

    Напряжение — это мера давления или силы электрической энергии, проходящей через проводник, тогда как ток — это, скорее, показатель скорости электрического потока.Это ток, проходящий через тело, сжимает сердце или вызывает его фибрилляцию, что может привести к смерти.

    Так что вопрос действительно должен быть: Сколько тока нужно, чтобы кого-то убить?

    Ответа очень мало. Сила тока всего 0,007 ампер (7 мА) через сердце в течение трех секунд достаточно, чтобы убить. Прохождение 0,1 ампер (100 мА) через тело почти наверняка приведет к летальному исходу.

    Однако сила тока при поражении электрическим током определяется напряжением и сопротивлением цепи.Человеческое тело обладает высоким сопротивлением электрическому току, что означает, что без достаточного напряжения опасное количество тока не может протекать через тело и вызывать травмы или смерть. Как показывает практика, более пятидесяти вольт достаточно, чтобы пропустить через тело потенциально смертельный ток.

    Другие факторы, которые могут определить степень поражения электрическим током, включают продолжительность удара и место его попадания в тело. Например, удар током, передаваемый от одной руки через грудь к другой руке, намного опаснее, чем удар между двумя пальцами ног.

    Вот несколько примеров:

    • Удар статическим электричеством может составлять 20000 вольт или более, но при очень низком токе и на очень короткое время: безвредно
    • Аккумулятор 9 В имеет недостаточное напряжение для прохождения опасного уровня тока через тело: безвредный
    • Розетка 240 В переменного тока находится под опасным напряжением и более чем способна пропускать очень опасный ток: потенциально смертельно опасный
    • Молния может иметь силу в миллиард вольт и может выдавать чрезвычайно высокий ток (около 30 000 ампер): потенциально смертельный исход

    Опасности поражения электрическим током и человеческое тело

    Цели обучения

    К концу этого раздела вы сможете:

    • Определите термическую опасность, опасность поражения электрическим током и короткого замыкания.
    • Объясните, какое влияние различные уровни тока оказывают на человеческое тело.

    Есть две известные опасности электричества — термическая и ударная. Термическая опасность — это опасность, при которой чрезмерная электрическая мощность вызывает нежелательные тепловые эффекты, такие как начало пожара в стене дома. Опасность поражения электрическим током возникает, когда электрический ток проходит через человека. Шок варьируется по степени тяжести от болезненного, но в остальном безвредного, до смертельного, вызывающего остановку сердца. В этом разделе количественно рассматриваются эти опасности и различные факторы, влияющие на них.Электробезопасность: Системы и устройства будут рассматривать системы и устройства для предотвращения поражения электрическим током.

    Электроэнергия вызывает нежелательные эффекты нагрева всякий раз, когда электрическая энергия преобразуется в тепловую со скоростью, превышающей ее безопасное рассеивание. Классическим примером этого является короткое замыкание , цепь с низким сопротивлением между выводами источника напряжения. Пример короткого замыкания показан на рисунке 1. Изоляция проводов, ведущих к прибору, изношена, что позволяет двум проводам соприкасаться.Такой нежелательный контакт с высоким напряжением называется коротким замыканием . Поскольку сопротивление короткого замыкания, r , очень мало, мощность, рассеиваемая коротким замыканием, P = В 2 / r , очень велика. Например, если В, составляет 120 В, а r составляет 0,100 Ом, тогда мощность составляет 144 кВт, что на намного больше, чем у обычного бытового прибора. Тепловая энергия, передаваемая с такой скоростью, очень быстро поднимет температуру окружающих материалов, плавя или, возможно, воспламеняя их.

    Рис. 1. Короткое замыкание — это нежелательный путь с низким сопротивлением через источник напряжения. (а) Изношенная изоляция проводов тостера позволяет им соприкасаться с низким сопротивлением r. Поскольку P = V 2 / r , тепловая энергия создается так быстро, что шнур плавится или горит. (б) Схема короткого замыкания.

    Один особенно коварный аспект короткого замыкания состоит в том, что его сопротивление может фактически уменьшиться из-за повышения температуры.Это может произойти, если короткое замыкание создает ионизацию. Эти заряженные атомы и молекулы могут свободно перемещаться и, таким образом, уменьшают сопротивление r . Поскольку P = В 2 / r , мощность, рассеиваемая при кратковременных повышениях, может вызвать большую ионизацию, большую мощность и т. Д. Высокое напряжение, такое как 480 В переменного тока, используемое в некоторых промышленных приложениях, поддается этой опасности, потому что более высокие напряжения создают более высокую начальную выработку энергии за короткое время.

    Другая серьезная, но менее драматическая тепловая опасность возникает, когда провода, по которым подается питание к пользователю, перегружены слишком большим током.Как обсуждалось в предыдущем разделе, мощность, рассеиваемая в проводах питания, составляет P = I 2 R w , где R w — сопротивление проводов, а I — сопротивление проводов. через них протекает ток. Если значение I или R w слишком велико, провода перегреваются. Например, изношенный шнур электроприбора (с порванной частью плетеной проволоки) может иметь R w = 2,00 Ом, а не 0.100 Ом должно быть. Если через шнур проходит ток 10,0 А, то в шнуре рассеивается P = I 2 R w = 200 Вт — намного больше, чем это безопасно. Точно так же, если провод с сопротивлением 0,100 Ом предназначен для передачи нескольких ампер, а вместо этого имеет ток 100 А, он сильно перегреется. Мощность, рассеиваемая в проводе, будет в этом случае P = 1000 Вт. Для ограничения чрезмерных токов используются предохранители и автоматические выключатели. (См. Рисунок 1 и рисунок 2.) Каждое устройство автоматически размыкает цепь, когда постоянный ток превышает безопасные пределы.

    Рис. 1. (a) Предохранитель имеет металлическую полосу с низкой температурой плавления, которая при перегреве чрезмерным током навсегда разрывает соединение цепи с источником напряжения. (b) Автоматический выключатель — это автоматический, но восстанавливаемый электрический выключатель. Показанный здесь имеет биметаллическую полосу, которая изгибается вправо и в выемку при перегреве. Затем пружина толкает металлическую полосу вниз, разрывая электрическое соединение в точках.

    Рис. 2. Схема цепи с предохранителем или автоматическим выключателем. Предохранители и автоматические выключатели действуют как автоматические выключатели, которые размыкаются, когда постоянный ток превышает желаемые пределы.

    Предохранители и автоматические выключатели для типичных бытовых напряжений и токов относительно просты в изготовлении, но предохранители для больших напряжений и токов имеют особые проблемы. Например, когда автоматический выключатель пытается прервать подачу высоковольтного электричества, через его точки может проскочить искра, которая ионизирует воздух в зазоре и позволяет току продолжать течь.В крупных автоматических выключателях, используемых в системах распределения электроэнергии, используется изолирующий газ и даже для гашения таких искр используются струи газа. Здесь переменный ток более безопасен, чем постоянный, поскольку переменный ток проходит через ноль 120 раз в секунду, что дает возможность быстро погасить эти дуги.

    Электрические токи, протекающие через людей, производят чрезвычайно разнообразные эффекты. Электрический ток можно использовать для блокирования боли в спине. Возможность использования электрического тока для стимуляции мышечной активности парализованных конечностей, что, возможно, позволит людям с параличом нижних конечностей ходить, изучается.Телевизионные драматизации, в которых электрические разряды используются, чтобы вывести жертву сердечного приступа из состояния фибрилляции желудочков (чрезвычайно нерегулярное, часто со смертельным исходом, сердцебиение), более чем обычны. Тем не менее, большинство смертельных случаев от поражения электрическим током происходит из-за того, что ток вызывает фибрилляцию сердца. Электрокардиостимулятор использует электрические разряды, чтобы заставить сердце биться правильно. Некоторые смертельные удары током не вызывают ожогов, но бородавки можно безопасно сжечь электрическим током (хотя сейчас более распространено замораживание с использованием жидкого азота).Конечно, этим разрозненным эффектам можно найти последовательные объяснения. Основными факторами, от которых зависят последствия поражения электрическим током, являются

    .
    1. Сумма тока I
    2. Путь, пройденный нынешними
    3. Продолжительность шока
    4. Частота f тока ( f = 0 для постоянного тока)

    В таблице 1 приведены эффекты поражения электрическим током в зависимости от тока для типичного случайного поражения электрическим током.Эффекты относятся к сотрясению, которое проходит через туловище, длится 1 с и вызывается мощностью 60 Гц.

    Рис. 3. Электрический ток может вызывать мышечные сокращения с различными эффектами. (а) Пострадавший «отбрасывается» назад из-за непроизвольных сокращений мышц, разгибающих ноги и туловище. (б) Пострадавший не может отпустить проволоку, которая стимулирует все мышцы руки. Смыкающие пальцы сильнее, чем разжимающие.

    Таблица 1.Эффекты поражения электрическим током в зависимости от силы тока
    Ток (мА) Эффект
    1 Порог ощущения
    5 Максимальный безопасный ток
    10–20 Начало устойчивого мышечного сокращения; не может отпустить на время шока; сокращение мышц груди может привести к остановке дыхания во время шока
    50 Начало боли
    100–300 + Возможна фибрилляция желудочков; часто со смертельным исходом
    300 Возникновение ожога в зависимости от концентрации тока
    6000 (6 А) Начало устойчивого сокращения желудочков и паралича дыхания; оба прекращаются, когда заканчивается шок; сердцебиение может вернуться в норму; используется для дефибрилляции сердца

    Наши тела являются относительно хорошими проводниками из-за воды в наших телах.Учитывая, что большие токи будут протекать через секции с меньшим сопротивлением (подробнее будет обсуждаться в следующей главе), электрические токи предпочтительно протекают по путям в человеческом теле, которые имеют минимальное сопротивление на прямом пути к земле. Земля — ​​естественный сток электронов. Ношение изолирующей обуви — требование во многих профессиях — препятствует прохождению электронов, создавая на этом пути большое сопротивление. При работе с мощными инструментами (сверлами) или в опасных ситуациях убедитесь, что вы не обеспечиваете путь для прохождения тока (особенно через сердце).

    Очень слабые токи проходят через тело безвредно и не чувствуются. Это происходит с вами регулярно без вашего ведома. Порог ощущения составляет всего 1 мА, и, несмотря на неприятные ощущения, разряды, по-видимому, безвредны для токов менее 5 мА. Во многих правилах безопасности значение 5 мА является максимально допустимым током. Ток от 10 до 20 мА и выше может стимулировать длительные мышечные сокращения так же, как обычные нервные импульсы. Иногда люди говорят, что они были сбиты с толку от шока, но на самом деле произошло то, что некоторые мышцы сократились, заставляя их двигаться не по их собственному выбору.(См. Рис. 3 (а).) Более пугающим и потенциально более опасным является эффект «не могу отпустить», проиллюстрированный на рис. 3 (б). Мышцы, закрывающие пальцы, сильнее, чем мышцы, открывающие их, поэтому рука непроизвольно смыкается на проводе, сотрясающем ее. Это может продлить шок на неопределенный срок. Это также может быть опасно для человека, пытающегося спасти жертву, потому что рука спасателя может сомкнуться на запястье жертвы. Обычно лучший способ помочь пострадавшему — это сильно ударить кулаком / ударом / встряхнуть изолятором или бросить изолятор в кулак.Современные электрические ограждения, используемые в вольерах для животных, теперь включаются и выключаются, чтобы люди, прикоснувшиеся к ним, могли освободиться, что делает их менее смертоносными, чем в прошлом.

    Сильные токи могут повлиять на сердце. Его электрические паттерны могут быть нарушены, так что он будет биться нерегулярно и неэффективно в состоянии, называемом «фибрилляция желудочков». Это состояние часто сохраняется после шока и приводит к летальному исходу из-за нарушения кровообращения. Порог фибрилляции желудочков составляет от 100 до 300 мА.При токе около 300 мА и выше разряд может вызвать ожоги, в зависимости от концентрации тока — чем более концентрированный, тем выше вероятность ожога.

    Очень большие токи заставляют сердце и диафрагму сокращаться на время разряда. И сердце, и дыхание останавливаются. Интересно, что оба часто возвращаются к нормальному состоянию после шока. Электрические паттерны в сердце полностью стираются, так что сердце может начать заново при нормальном биении, в отличие от постоянного нарушения, вызванного меньшими токами, которые могут вызвать фибрилляцию желудочков в сердце.Последнее похоже на каракули на доске, тогда как первое полностью стирает его. В телесериалах о поражении электрическим током, используемом для выведения жертвы сердечного приступа из состояния фибрилляции желудочков, также показаны большие лопасти. Они используются для распределения тока, проходящего через пострадавшего, чтобы снизить вероятность ожогов.

    Ток является основным фактором, определяющим серьезность удара (при условии, что другие условия, такие как путь, продолжительность и частота, являются фиксированными, например, в таблице и в предыдущем обсуждении).Более высокое напряжение более опасно, но, поскольку I = V / R , сила удара зависит от комбинации напряжения и сопротивления. Например, у человека с сухой кожей сопротивление составляет около 200 кОм. Если он соприкасается с 120-В переменного тока, через него безвредно проходит ток I = (120 В) / (200 кОм) = 0,6 мА. Тот же человек, намоченный насквозь, может иметь сопротивление 10,0 кОм, и те же 120 В будут производить ток 12 мА — выше порога «не отпускать» и потенциально опасен.

    Большая часть сопротивления тела находится в его сухой коже. Во влажном состоянии соли переходят в ионную форму, что значительно снижает сопротивление. Внутренняя часть тела имеет гораздо меньшее сопротивление, чем сухая кожа, из-за всех содержащихся в ней ионных растворов и жидкостей. Если обойти сопротивление кожи, например, с помощью внутривенной инфузии, катетера или открытого электрокардиостимулятора, человек становится чувствительным к микрошоку . В этом состоянии токи около 1/1000 от перечисленных в таблице 1 производят аналогичные эффекты.Во время операции на открытом сердце можно использовать ток до 20 мкА, чтобы успокоить сердце. Строгие требования к электробезопасности в больницах, особенно в хирургии и реанимации, связаны с вдвойне менее уязвимыми пациентами, чувствительными к микрошоку. Разрыв кожи уменьшил его сопротивление, поэтому одно и то же напряжение вызывает больший ток, а гораздо меньший ток имеет больший эффект.

    Рис. 4. График средних значений порога ощущения и тока «не могу отпустить» в зависимости от частоты.Чем ниже значение, тем более чувствительно тело к этой частоте.

    Другими факторами, кроме силы тока, которые влияют на серьезность разряда, являются его путь, продолжительность и частота переменного тока. Путь имеет очевидные последствия. Например, сердце не поражается электрическим током через мозг, который может использоваться для лечения маниакальной депрессии. И это общая правда, что чем больше продолжительность шока, тем сильнее его последствия. На рисунке 4 представлен график, иллюстрирующий влияние частоты на ударную нагрузку.Кривые показывают минимальный ток для двух различных эффектов как функцию частоты. Чем ниже необходимый ток, тем чувствительнее тело к этой частоте. По иронии судьбы, тело наиболее чувствительно к частотам, близким к обычным частотам 50 или 60 Гц. Тело немного менее чувствительно к постоянному току ( f = 0), что мягко подтверждает утверждения Эдисона о том, что переменный ток представляет большую опасность. На все более высоких частотах организм становится все менее чувствительным к любым воздействиям, затрагивающим нервы.Это связано с максимальной скоростью, с которой нервы могут активироваться или стимулироваться. Электрический ток на очень высоких частотах распространяется только по поверхности человека. Таким образом, бородавку можно сжечь током очень высокой частоты, не вызывая остановки сердца. (Не пытайтесь делать это дома с переменным током 60 Гц!) Некоторые из зрелищных демонстраций электричества, в которых дуги высокого напряжения проходят по воздуху и над телами людей, используют высокие частоты и малые токи. (См. Рисунок 5.) Устройства и методы электробезопасности подробно описаны в разделе «Электробезопасность: системы и устройства».

    Рис. 5 Опасна ли эта электрическая дуга? Ответ зависит от частоты переменного тока и мощности. (Источник: Химич Алекс, Wikimedia Commons)

    Сводка раздела

    • Существует два типа опасности поражения электрическим током: термическое (чрезмерная мощность) и поражение электрическим током (электрический ток через человека).
    • Сила удара определяется силой тока, длиной пути, продолжительностью и частотой переменного тока.
    • В таблице 1 перечислены опасности поражения электрическим током в зависимости от силы тока.
    • На рис. 5 показан график порогового тока для двух опасностей в зависимости от частоты.

    Концептуальные вопросы

    1. С помощью омметра ученик измеряет сопротивление между различными точками своего тела. Он обнаружил, что сопротивление между двумя точками на одном пальце примерно такое же, как сопротивление между двумя точками на противоположных руках — обе составляют несколько сотен тысяч Ом. Кроме того, сопротивление уменьшается, когда большее количество кожи контактирует с щупами омметра. Наконец, наблюдается резкое падение сопротивления (до нескольких тысяч Ом), когда кожа влажная.Объясните эти наблюдения и их значение для кожи и внутреннего сопротивления человеческого тела.
    2. Каковы две основные опасности электричества?
    3. Почему короткое замыкание не создает опасности поражения электрическим током?
    4. От чего зависит тяжесть шока? Можете ли вы сказать, что определенное напряжение опасно, без дополнительной информации?
    5. Электрифицированная игла используется для выжигания бородавок, при этом цепь замыкается путем усаживания пациента на большую пластину приклада.Почему эта тарелка большая?
    6. Некоторые операции выполняются при прохождении электричества высокого напряжения от металлического скальпеля через разрезаемую ткань. Учитывая природу электрических полей на поверхности проводников, почему вы ожидаете, что большая часть тока будет течь от острого края скальпеля? Как вы думаете, используется переменный ток высокой или низкой частоты?
    7. На некоторых устройствах, которые часто используются в ванных комнатах, например, в фенах, есть сообщения о безопасности, в которых говорится: «Не используйте, когда ванна или раковина наполнены водой.«Почему это так?
    8. Нам часто советуют не щелкать электрическими выключателями мокрыми руками, сначала вытереть руки. Также рекомендуется никогда не поливать электрический огонь водой. Почему это так?
    9. Перед тем, как приступить к работе на линии электропередачи, линейные монтеры будут касаться линии тыльной стороной руки в качестве окончательной проверки нулевого напряжения. Почему тыльная сторона руки?
    10. Почему сопротивление влажной кожи намного меньше, чем сопротивление сухой, и почему кровь и другие жидкости организма имеют низкое сопротивление?
    11. Может ли человек, получающий внутривенное вливание (в / в) быть чувствительным к микрошоку?
    12. Принимая во внимание небольшие токи, которые вызывают опасность поражения электрическим током, и большие токи, которые прерывают автоматические выключатели и предохранители, как они играют роль в предотвращении опасности поражения электрическим током?

    Задачи и упражнения

    1.(a) Сколько мощности рассеивается при коротком замыкании 240 В переменного тока через сопротивление 0,250 Ом? б) Какой ток течет?

    2. Какое напряжение возникает при коротком замыкании 1,44 кВт через сопротивление 0,100 Ом?

    3. Найдите ток, протекающий через человека, и определите вероятное воздействие на него, если он коснется источника переменного тока напряжением 120 В: (а) если он стоит на резиновом коврике и предлагает полное сопротивление 300 кОм; (б) если она стоит босиком на мокрой траве и имеет сопротивление всего 4000 кОм.

    4. Принимая ванну, человек касается металлического корпуса радиоприемника. Путь через человека к водосточной трубе и земле имеет сопротивление 4000 Ом. Какое наименьшее напряжение на корпусе радио может вызвать фибрилляцию желудочков?

    5. Глупо пытаясь выудить горящий кусок хлеба из тостера металлическим ножом для масла, человек контактирует с напряжением 120 В переменного тока. Он даже не чувствует этого, потому что, к счастью, на нем туфли на резиновой подошве. Какое минимальное сопротивление пути, по которому ток проходит через человека?

    6.(а) Во время операции ток величиной всего 20,0 мкА, приложенный непосредственно к сердцу, может вызвать фибрилляцию желудочков. Если сопротивление обнаженного сердца составляет 300 Ом, какое наименьшее напряжение представляет эту опасность? (b) Подразумевает ли ваш ответ, что необходимы особые меры предосторожности в отношении электробезопасности?

    7. (a) Каково сопротивление короткого замыкания 220 В переменного тока, которое генерирует пиковую мощность 96,8 кВт? (b) Какой была бы средняя мощность, если бы напряжение составляло 120 В переменного тока?

    8.Дефибриллятор сердца пропускает 10,0 А через туловище пациента в течение 5,00 мс в попытке восстановить нормальное сердцебиение. а) Сколько заряда прошло? (б) Какое напряжение было приложено, если было рассеяно 500 Дж энергии? (c) Какое сопротивление было на пути? (d) Найдите повышение температуры в 8,00 кг пораженной ткани.

    9. Integrated Concepts Короткое замыкание в шнуре электроприбора на 120 В имеет сопротивление 0,500 Ом. Рассчитайте превышение температуры 2,00 г окружающих материалов, принимая их удельную теплоемкость равной 0.200 кал / г ºC и что автоматическому выключателю требуется 0,0500 с для отключения тока. Это может быть опасно?

    10. Температура увеличивается на 860ºC. Очень вероятно, что это повредит.

    11. Создайте свою проблему Представьте себе человека, работающего в среде, где электрические токи могут проходить через ее тело. Постройте задачу, в которой вы рассчитываете сопротивление изоляции, необходимое для защиты человека от повреждений. Среди вещей, которые следует учитывать, — напряжение, которому может подвергнуться человек, вероятное сопротивление тела (сухой, влажный,…) и допустимые токи (безопасные, но ощутимые, безопасные и неощутимые,…).

    Глоссарий

    термическая опасность:
    опасность, при которой электрический ток вызывает нежелательные тепловые эффекты
    опасность поражения электрическим током:
    при прохождении электрического тока через человека
    короткое замыкание:
    , ​​также известный как «короткий» путь с низким сопротивлением между выводами источника напряжения
    чувствительность к микрошоку:
    состояние, при котором сопротивление кожи человека обходится, возможно, с помощью медицинской процедуры, что делает человека уязвимым для поражения электрическим током при токах около 1/1000 от обычно необходимого уровня

    Избранные решения проблем и упражнения

    1.(а) 230 кВт (б) 960 А

    3. (а) 0,400 мА, нет эффекта (б) 26,7 мА, мышечное сокращение на время шока (не могу отпустить)

    5. 1,20 × 10 5 Ом

    7. (а) 1,00 Ом (б) 14,4 кВт


    ЭЛЕКТРИЧЕСКАЯ БЕЗОПАСНОСТЬ — прикладное промышленное электричество

    Важность электробезопасности

    С помощью этого урока я надеюсь избежать распространенной ошибки, обнаруживаемой в учебниках по электронике, состоящей в игнорировании или недостаточном освещении темы электробезопасности.Я предполагаю, что тот, кто читает эту книгу, хотя бы мимолетно заинтересован в реальной работе с электричеством, и поэтому тема безопасности имеет первостепенное значение.

    Еще одно преимущество включения подробного урока по электробезопасности — это практический контекст, который он устанавливает для основных понятий напряжения, тока, сопротивления и проектирования схем. Чем более актуальной будет техническая тема, тем больше вероятность того, что студент обратит внимание и поймет. А что может быть важнее приложения для личной безопасности? Кроме того, поскольку электрическая энергия является повседневным явлением в современной жизни, почти каждый может ознакомиться с иллюстрациями, приведенными на таком уроке.Вы когда-нибудь задумывались, почему птиц не шокирует, когда они отдыхают на линиях электропередач? Читайте и узнайте!

    Физиологические эффекты электричества

    Большинство из нас испытали ту или иную форму электрического «шока», когда электричество заставляет наше тело испытывать боль или травму. Если нам повезет, степень этого переживания ограничится покалыванием или приступами боли от накопления статического электричества, разряженного через наши тела. Когда мы работаем с электрическими цепями, способными передавать большую мощность нагрузкам, поражение электрическим током становится гораздо более серьезной проблемой, а боль — наименее значимым результатом поражения электрическим током.

    Поскольку электрический ток проходит через материал, любое противодействие току (сопротивлению) приводит к рассеиванию энергии, обычно в виде тепла. Это самый простой и понятный эффект воздействия электричества на живую ткань: ток заставляет ее нагреваться. Если количество выделяемого тепла достаточно, ткань может обжечься. Эффект носит физиологический характер, такой же, как повреждение, вызванное открытым пламенем или другим высокотемпературным источником тепла, за исключением того, что электричество обладает способностью сжигать ткани под кожей жертвы, даже обжигая внутренние органы.

    Как электрический ток влияет на нервную систему

    Еще одно воздействие электрического тока на организм, возможно, наиболее опасное, касается нервной системы. Под «нервной системой» я имею в виду сеть особых клеток в организме, называемых нервными клетками или нейронами, которые обрабатывают и проводят множество сигналов, ответственных за регуляцию многих функций организма. Мозг, спинной мозг и сенсорные / двигательные органы в теле функционируют вместе, позволяя ему чувствовать, двигаться, реагировать, думать и запоминать.

    Нервные клетки взаимодействуют друг с другом, действуя как «преобразователи», создавая электрические сигналы (очень малые напряжения и токи) в ответ на ввод определенных химических соединений, называемых нейротрансмиттерами , и высвобождая эти нейротрансмиттеры при стимуляции электрическими сигналами. Если электрический ток достаточной силы проходит через живое существо (человека или другое), его эффектом будет подавление крошечных электрических импульсов, обычно генерируемых нейронами, перегрузка нервной системы и предотвращение способности рефлекторных и волевых сигналов действовать. задействовать мышцы.Мышцы, вызванные внешним (шоковым) током, непроизвольно сокращаются, и жертва ничего не может с этим поделать.

    Эта проблема особенно опасна, если пострадавший касается руками проводника под напряжением. Мышцы предплечья, отвечающие за сгибание пальцев, как правило, лучше развиты, чем мышцы, отвечающие за разгибание пальцев, и поэтому, если оба набора мышц будут пытаться сокращаться из-за электрического тока, проводимого через руку человека, «сгибающие» мышцы выиграют, сжимая пальцы в кулак.Если проводник, подающий ток к пострадавшему, обращен к ладони его или ее руки, это сжимающее действие заставит руку крепко ухватиться за провод, тем самым ухудшив ситуацию, обеспечивая отличный контакт с проводом. Пострадавший совершенно не сможет отпустить проволоку.

    С медицинской точки зрения это состояние непроизвольного сокращения мышц называется столбняк . Электрики, знакомые с этим эффектом поражения электрическим током, часто называют обездвиженную жертву поражения электрическим током «зависшей в цепи».Вызванный током столбняк можно прервать, только отключив ток через пострадавшего.

    Даже когда ток прекращается, жертва не может восстановить произвольный контроль над своими мышцами в течение некоторого времени, так как химический состав нейротрансмиттера находится в беспорядке. Этот принцип был применен в устройствах «электрошокера», таких как электрошокеры, которые основаны на принципе мгновенного поражения жертвы высоковольтным импульсом, передаваемым между двумя электродами. Правильно нанесенный электрошокер временно (на несколько минут) обездвиживает жертву.

    Однако электрический ток может воздействовать не только на скелетные мышцы жертвы электрошока. Мышца диафрагмы, контролирующая легкие, и сердце, которое само по себе является мышцей, также могут быть «заморожены» в состоянии столбняка электрическим током. Даже токи, слишком слабые для того, чтобы вызвать столбняк, часто способны перебивать сигналы нервных клеток настолько, что сердце не может биться должным образом, что приводит к состоянию, известному как фибрилляция . Фибриллирующее сердце скорее трепещет, чем бьется, и не может перекачивать кровь к жизненно важным органам тела.В любом случае смерть от удушья и / или остановки сердца обязательно наступит из-за достаточно сильного электрического тока, проходящего через тело. По иронии судьбы, медицинский персонал использует сильный разряд электрического тока, прикладываемый к груди жертвы, чтобы «подтолкнуть» фибриллирующее сердце к нормальному ритму биений.

    Эта последняя деталь подводит нас к другой опасности поражения электрическим током, которая свойственна общественным энергосистемам. Хотя наше первоначальное исследование электрических цепей будет сосредоточено почти исключительно на постоянном токе (постоянном токе или электричестве, которое движется в непрерывном направлении в цепи), современные энергетические системы используют переменный ток или переменный ток.Технические причины такого предпочтения переменного тока перед постоянным током в энергосистемах не имеют отношения к этому обсуждению, но особые опасности каждого вида электроэнергии очень важны для темы безопасности.

    Воздействие переменного тока на организм во многом зависит от частоты. Низкочастотный (от 50 до 60 Гц) переменный ток используется в домашних хозяйствах США (60 Гц) и Европы (50 Гц); он может быть опаснее высокочастотного переменного тока и в 3-5 раз опаснее постоянного тока того же напряжения и силы тока. Низкочастотный переменный ток вызывает длительное сокращение мышц (тетанию), которое может прижать руку к источнику тока, продлевая воздействие.Постоянный ток, скорее всего, вызовет одиночное судорожное сокращение, которое часто заставляет жертву отойти от источника тока.

    Переменный характер

    AC имеет большую тенденцию приводить нейроны кардиостимулятора в состояние фибрилляции, тогда как DC имеет тенденцию просто вызывать остановку сердца. Как только ток разряда прекращается, у «замороженного» сердца больше шансов восстановить нормальный ритм сердечных сокращений, чем у фибриллирующего сердца. Вот почему «дефибриллирующее» оборудование, используемое врачами скорой помощи, работает: электрический разряд, подаваемый дефибриллятором, — это постоянный ток, который останавливает фибрилляцию и дает сердцу шанс восстановиться.

    В любом случае электрические токи, достаточно высокие, чтобы вызвать непроизвольное мышечное действие, опасны, и их следует избегать любой ценой. В следующем разделе мы рассмотрим, как такие токи обычно входят в тело и выходят из него, а также рассмотрим меры предосторожности против таких случаев.

    • Электрический ток может вызвать глубокие и серьезные ожоги тела из-за рассеяния мощности через электрическое сопротивление тела.
    • Столбняк — это состояние, при котором мышцы непроизвольно сокращаются из-за прохождения внешнего электрического тока через тело.Когда непроизвольное сокращение мышц, управляющих пальцами, приводит к тому, что жертва не может отпустить проводник, находящийся под напряжением, жертва считается «замороженной в цепи».
    • Диафрагма (легкие) и сердечные мышцы одинаково подвержены воздействию электрического тока. Даже токи, слишком слабые, чтобы вызвать столбняк, могут быть достаточно сильными, чтобы мешать работе нейронов кардиостимулятора, заставляя сердце трепетать, а не сильно биться.
    • Постоянный ток (DC) с большей вероятностью может вызвать столбняк в мышцах, чем переменный ток (AC), поэтому постоянный ток с большей вероятностью «заморозит» жертву в случае шока.Однако переменный ток с большей вероятностью вызовет фибрилляцию сердца жертвы, что является более опасным состоянием для жертвы после прекращения действия электрического тока.

    Электричество требует полного пути (цепи) для непрерывного потока. Вот почему удар, полученный от статического электричества, является только мгновенным толчком: течение тока обязательно кратковременно, когда статические заряды уравниваются между двумя объектами. Подобные шоки самоограниченной продолжительности редко бывают опасными.

    Без двух точек контакта на теле для входа и выхода тока, соответственно, опасность поражения электрическим током отсутствует. Вот почему птицы могут спокойно отдыхать на высоковольтных линиях электропередачи, не подвергаясь электрошоку: они контактируют с цепью только в одной точке.

    Рис. 1.1

    Для того, чтобы ток протекал по проводнику, должно присутствовать напряжение, которое его мотивирует. Напряжение, как вы должны помнить, всегда составляет относительно двух точек . Нет такой вещи, как напряжение «на» или «в» одной точке цепи, и поэтому птица, контактирующая с одной точкой в ​​вышеуказанной цепи, не имеет напряжения, приложенного к ее телу, чтобы установить ток через нее.Да, даже если они опираются на , два футов, обе ноги касаются одного и того же провода, что делает их электрически общими . С точки зрения электричества, обе птичьи лапы соприкасаются с одной и той же точкой, поэтому между ними нет напряжения, которое могло бы стимулировать ток через тело птицы.

    Это может привести к мысли, что невозможно получить поражение электрическим током, прикоснувшись только к одному проводу. Как птицы, если мы будем касаться только одного провода за раз, мы будем в безопасности, верно? К сожалению, это не так.В отличие от птиц, при контакте с «живым» проводом люди обычно стоят на земле. Часто одна сторона энергосистемы будет намеренно подключена к заземлению, и поэтому человек, касающийся одиночного провода, фактически устанавливает контакт между двумя точками в цепи (провод и заземление):

    Рис. 1.2

    Значок земли представляет собой набор из трех горизонтальных полос уменьшающейся ширины, расположенных в нижнем левом углу показанной схемы, а также у ступни человека, подвергающегося электрошоку.В реальной жизни заземление энергосистемы представляет собой какой-то металлический проводник, закопанный глубоко в землю для обеспечения максимального контакта с землей. Этот проводник электрически подключен к соответствующей точке соединения в цепи толстым проводом. Заземление жертвы осуществляется через ноги, которые касаются земли.

    В этот момент в уме ученика обычно возникает несколько вопросов:

    • Если наличие точки заземления в цепи обеспечивает легкую точку контакта для кого-то, чтобы получить удар током, зачем вообще она в цепи? Разве схема без заземления не была бы безопаснее?
    • Человек, которого шокирует, вероятно, не ходит босиком.Если резина и ткань являются изоляционными материалами, то почему их обувь не защищает их, предотвращая образование цепи?
    • Насколько хорошим проводником может быть грязь ? Если вы можете быть поражены током, протекающим через землю, почему бы не использовать землю в качестве проводника в наших силовых цепях?

    В ответ на первый вопрос, наличие преднамеренной точки «заземления» в электрической цепи предназначено для обеспечения того, чтобы одна сторона ее была безопасна для контакта.Обратите внимание, что если бы наша жертва на приведенной выше диаграмме коснулась нижней стороны резистора, ничего бы не произошло, даже если бы их ноги все еще касались земли:

    Рис. 1.3.

    Поскольку нижняя сторона схемы надежно соединена с землей через точку заземления в нижнем левом углу схемы, нижний проводник схемы имеет электрически общий вид с заземлением. Поскольку между электрически общими точками не может быть напряжения, на человека, контактирующего с нижним проводом, не будет напряжения, и он не получит удара током.По той же причине провод, соединяющий цепь с заземляющим стержнем / пластинами, обычно остается оголенным (без изоляции), так что любой металлический объект, о который он задевает, будет электрически общим с землей.

    Заземление цепи гарантирует, что по крайней мере одна точка в цепи будет безопасна для прикосновения. Но как насчет того, чтобы оставить цепь полностью незаземленной? Разве это не сделало бы человека, касающегося только одного провода, таким же безопасным, как птица, сидящая только на одном? В идеале да. Практически нет.Посмотрите, что происходит без земли:

    Рисунок 1.4

    Несмотря на то, что ноги человека все еще соприкасаются с землей, любая точка в цепи должна быть безопасной для прикосновения. Поскольку не существует полного пути (цепи), проходящего через тело человека от нижней стороны источника напряжения к верхней, нет возможности установить ток через человека. Однако все это может измениться из-за случайного заземления, например, если ветка дерева касается линии электропередачи и обеспечивает соединение с землей.Такое случайное соединение между проводником энергосистемы и землей называется замыканием на землю .

    Рисунок 1.5

    Замыкания на землю

    Замыкания на землю могут быть вызваны многими причинами, в том числе скоплением грязи на изоляторах линий электропередач (создание пути грязной воды для тока от проводника к полюсу и к земле во время дождя), проникновением грунтовых вод в подземные проводники линии электропередач. , и птицы, приземляющиеся на линии электропередачи, перемыкая линию к полюсу своими крыльями.Учитывая множество причин замыканий на землю, они, как правило, непредсказуемы. В случае с деревьями никто не может гарантировать , с каким проводом могут соприкасаться их ветви. Если бы дерево задело верхний провод в цепи, это сделало бы верхний провод безопасным для прикосновения, а нижний опасным — как раз противоположность предыдущему сценарию, когда дерево касается нижнего провода:

    Рисунок 1.6

    Когда ветвь дерева соприкасается с верхним проводом, этот провод становится заземленным проводом в цепи, электрически общим с заземлением.Следовательно, между этим проводом и землей нет напряжения, а есть полное (высокое) напряжение между нижним проводом и землей. Как упоминалось ранее, ветви деревьев являются лишь одним потенциальным источником замыканий на землю в энергосистеме. Рассмотрим незаземленную энергосистему без соприкосновения деревьев с деревьями, но на этот раз с двумя людьми, касающимися одиночных проводов:

    Рис. 1.7

    Когда каждый человек стоит на земле, контактируя с разными точками цепи, путь электрического тока проходит через одного человека, через землю и через другого человека.Несмотря на то, что каждый человек думает, что он в безопасности, только коснувшись одной точки в цепи, их совместные действия создают смертельный сценарий. Фактически, один человек действует как замыкание на землю, что делает его небезопасным для другого человека. Именно поэтому незаземленные энергосистемы опасны: напряжение между любой точкой цепи и землей (землей) непредсказуемо, потому что замыкание на землю может возникнуть в любой точке цепи в любое время. Единственный персонаж, который гарантированно будет в безопасности в этих сценариях, — это птица, которая вообще не связана с землей! Надежно подключив обозначенную точку цепи к заземлению («заземлив» цепь), по крайней мере, безопасность может быть обеспечена в этой точке.Это большая гарантия безопасности, чем полное отсутствие заземления.

    Отвечая на второй вопрос, обувь с резиновой подошвой действительно обеспечивает некоторую электрическую изоляцию, чтобы помочь защитить кого-то от проведения электрического тока через ступни. Однако наиболее распространенные конструкции обуви не являются электрически «безопасными», поскольку их подошва слишком тонкая и не из подходящего материала. Кроме того, любая влага, грязь или токопроводящие соли из пота тела на поверхности подошвы или проникающие через нее могут поставить под угрозу ту небольшую изоляционную ценность, которая должна была изначально иметь обувь.Есть обувь, специально предназначенная для опасных электромонтажных работ, а также толстые резиновые коврики, на которых можно стоять во время работы с цепями под напряжением, но эти специальные детали должны быть в абсолютно чистом и сухом состоянии, чтобы быть эффективными. Достаточно сказать, что обычной обуви недостаточно, чтобы гарантировать защиту от поражения электрическим током от электросети.

    Исследования контактного сопротивления между частями человеческого тела и точками контакта (например, с землей) показывают широкий диапазон цифр (информацию об источнике этих данных см. В конце главы):

    • Контакт для рук или ног, с резиновой изоляцией: обычно 20 МОм.
    • Контакт ступни через кожаную подошву обуви (сухой): от 100 кОм до 500 кОм
    • Контакт ступни через кожаную подошву обуви (мокрая): от 5 кОм до 20 кОм

    Как видите, резина не только является гораздо лучшим изоляционным материалом, чем кожа, но и присутствие воды в пористом веществе, таком как кожа , значительно снижает электрическое сопротивление.

    Отвечая на третий вопрос, грязь — не очень хороший проводник (по крайней мере, когда она сухая!). У него слишком плохой проводник, чтобы поддерживать постоянный ток для питания нагрузки.Однако, как мы увидим в следующем разделе, требуется очень мало тока, чтобы ранить или убить человека, поэтому даже плохой проводимости грязи достаточно, чтобы обеспечить путь для смертельного тока при наличии достаточного напряжения, как обычно находится в энергосистемах.

    Некоторые шлифованные поверхности лучше изолируют, чем другие. Например, асфальт на масляной основе имеет гораздо большее сопротивление, чем большинство видов грязи или камней. Бетон, с другой стороны, имеет довольно низкое сопротивление из-за внутреннего содержания воды и электролита (проводящего химического вещества).

    • Поражение электрическим током может произойти только при контакте между двумя точками цепи; когда на тело жертвы подается напряжение.
    • Цепи питания
    • обычно имеют обозначенную точку, которая «заземлена»: прочно соединена с металлическими стержнями или пластинами, закопанными в грязь, чтобы гарантировать, что одна сторона цепи всегда находится под потенциалом земли (нулевое напряжение между этой точкой и землей).
    • Замыкание на землю — это случайное соединение проводника цепи с землей (землей).
    • Специальная изолированная обувь и коврики предназначены для защиты людей от ударов через заземление, но даже эти части снаряжения должны быть в чистом, сухом состоянии, чтобы быть эффективными. Обычная обувь недостаточно хороша, чтобы обеспечить защиту от ударов, изолируя ее владельца от земли.
    • Хотя грязь — плохой проводник, она может проводить достаточно тока, чтобы ранить или убить человека.

    Распространенная фраза в отношении электробезопасности звучит примерно так: « Убивает не напряжение, а ток ! ”Хотя в этом есть доля правды, об опасности поражения электрическим током нужно понимать больше, чем эта простая пословица.Если бы напряжение не представляло опасности, никто бы никогда не распечатал и не вывесил надписи: ОПАСНО — ВЫСОКОЕ НАПРЯЖЕНИЕ!

    Принцип «убивает текущее» по сути верен. Это электрический ток, который сжигает ткани, замораживает мышцы и вызывает фибрилляцию сердца. Однако электрический ток не возникает сам по себе: должно быть доступное напряжение, чтобы побудить ток протекать через жертву. Тело человека также оказывает сопротивление току, что необходимо учитывать.

    Принимая закон Ома для напряжения, тока и сопротивления и выражая его через ток для заданных напряжения и сопротивления, мы получаем следующее уравнение:

    [латекс] \ textbf {закон Ома} [/ латекс]

    [латекс] Ток = \ frac {Напряжение} {Сопротивление} [/ латекс] [латекс] I = \ frac {E} {R} [/ латекс]

    Величина тока, протекающего через тело, равна величине напряжения, приложенного между двумя точками этого тела, деленному на электрическое сопротивление, оказываемое телом между этими двумя точками.Очевидно, что чем больше напряжения доступно для протекания тока, тем легче он будет проходить через любое заданное сопротивление. Следовательно, существует опасность высокого напряжения, которое может генерировать ток, достаточный для получения травмы или смерти. И наоборот, если тело имеет более высокое сопротивление, меньший ток будет протекать при любом заданном напряжении. Насколько опасно напряжение, зависит от общего сопротивления цепи, препятствующего прохождению электрического тока.

    Сопротивление тела не является фиксированной величиной.Это варьируется от человека к человеку и время от времени. Существует даже метод измерения содержания жира в организме, основанный на измерении электрического сопротивления между пальцами рук и ног. Различное процентное содержание жира в организме обеспечивает разное сопротивление: одна переменная влияет на электрическое сопротивление в организме человека. Чтобы методика работала точно, человек должен регулировать потребление жидкости за несколько часов до теста, что указывает на то, что гидратация тела является еще одним фактором, влияющим на электрическое сопротивление тела.

    Сопротивление тела также варьируется в зависимости от того, как происходит контакт с кожей: от руки к руке, от руки к ноге, от ступни к ступне, от руки к локтю и т. Д. Пот, богатый солью и минералами. , являясь жидкостью, является отличным проводником электричества. То же самое и с кровью с таким же высоким содержанием проводящих химикатов. Таким образом, контакт с проводом потной рукой или открытой раной будет оказывать гораздо меньшее сопротивление току, чем контакт с чистой сухой кожей.

    Измеряя электрическое сопротивление чувствительным измерителем, я измеряю примерно 1 миллион Ом (1 МОм) на руках, держась за металлические щупы измерителя между пальцами.Измеритель показывает меньшее сопротивление, когда я крепко сжимаю щупы, и большее сопротивление, когда я держу их свободно. Я сижу за компьютером и печатаю эти слова, мои руки чистые и сухие. Если бы я работал в жаркой, грязной промышленной среде, сопротивление между моими руками, вероятно, было бы намного меньше, представляя меньшее сопротивление смертельному току и большую опасность поражения электрическим током.

    Насколько опасен электрический ток?

    Ответ на этот вопрос также зависит от нескольких факторов.Химический состав тела человека оказывает значительное влияние на то, как электрический ток влияет на человека. Некоторые люди очень чувствительны к току, испытывая непроизвольное сокращение мышц из-за ударов статического электричества. Другие могут получить большие искры от разряда статического электричества и почти не почувствовать его, не говоря уже о мышечном спазме. Несмотря на эти различия, с помощью тестов были разработаны приблизительные руководящие принципы, которые показывают, что для проявления вредных эффектов требуется очень небольшой ток (опять же, информацию об источнике этих данных см. В конце главы).Все текущие значения даны в миллиамперах (миллиампер равен 1/1000 ампер):

    ТЕЛО ВЛИЯНИЕ МУЖЧИНЫ / ЖЕНЩИНЫ ПРЯМОЙ ТОК (DC) 60 Гц 100 кГц
    Легкое ощущение под рукой Мужчины 1,0 мА 0,4 мА 7 мА
    Женщины 0,6 мА 0,3 мА 5 мА
    Порог боли Мужчины 5.2 мА 1,1 мА 12 мА
    Женщины 3,5 мА 0,7 мА 8 мА
    Болезненный, но произвольный контроль мышц сохраняется Мужчины 62 мА 9 мА 55 мА
    Женщины 41 мА 6 мА 37 мА
    Болезненно, провода не отпускаются Мужчины 76 мА 16 мА 75 мА
    Женщины 60 мА 15 мА 63 мА
    Сильная боль, затрудненное дыхание Мужчины 90 мА 23 мА 94 мА
    Женщины 60 мА 15 мА 63 мА
    Возможная фибрилляция сердца через 3 секунды Мужчины и женщины 500 мА 100 мА

    «Гц» означает единицу измерения Гц .Это мера того, насколько быстро меняется переменный ток, иначе известный как частота . Таким образом, столбец цифр, обозначенный «60 Гц переменного тока», относится к току, который меняется с частотой 60 циклов (1 цикл = период времени, когда ток течет в одном направлении, а затем в другом) в секунду. Последний столбец, обозначенный «10 кГц переменного тока», относится к переменному току, который совершает десять тысяч (10 000) возвратно-поступательных циклов каждую секунду.

    Имейте в виду, что эти цифры являются приблизительными, поскольку люди с различным химическим составом тела могут реагировать по-разному.Было высказано предположение, что ток через грудную клетку всего 17 мА переменного тока достаточно, чтобы вызвать фибрилляцию у человека при определенных условиях. Большинство наших данных относительно индуцированной фибрилляции получены в результате испытаний на животных. Очевидно, что проводить тесты индуцированной фибрилляции желудочков на людях непрактично, поэтому имеющиеся данные отрывочны. О, и если вам интересно, я понятия не имею, почему женщины, как правило, более восприимчивы к электрическому току, чем мужчины! Предположим, я положил руки на клеммы источника переменного напряжения с частотой 60 Гц (60 циклов в секунду).Какое напряжение необходимо для этого состояния чистой, сухой кожи, чтобы получить ток в 20 миллиампер (достаточно, чтобы я не мог отпустить источник напряжения)? Мы можем использовать закон Ома, чтобы определить это:

    [латекс] E = IR [/ латекс]

    [латекс] E = (20 мА) (1 M \ Omega) [/ латекс]

    [латекс] \ textbf {E = 20 000 вольт или 20 кВ} [/ латекс]

    Имейте в виду, что это «лучший случай» (чистая, сухая кожа) с точки зрения электробезопасности и что это значение напряжения представляет собой величину, необходимую для индукции столбняка.Чтобы вызвать болезненный шок, потребуется гораздо меньше! Кроме того, имейте в виду, что физиологические эффекты любой конкретной силы тока могут значительно отличаться от человека к человеку, и что эти расчеты являются приблизительными оценками , всего лишь .

    Обрызгав пальцы водой для имитации пота, я смог измерить сопротивление рук в руках всего 17 000 Ом (17 кОм). Имейте в виду, что это касается только одного пальца каждой руки, касающегося тонкой металлической проволоки. Пересчитав напряжение, необходимое для возникновения тока в 20 мА, мы получим эту цифру:

    [латекс] E = IR [/ латекс]

    [латекс] E = (20 мА) (17 кОмега) [/ латекс]

    [латекс] \ textbf {E = 340 V} [/ латекс]

    В этих реальных условиях потребуется всего 340 вольт потенциала от одной моей руки к другой, чтобы вызвать ток 20 миллиампер.Тем не менее, все же возможно получить смертельный удар от меньшего напряжения, чем это. При условии значительно более низкого показателя сопротивления тела, увеличенного за счет контакта с кольцом (полоса золота, обернутая по окружности пальца, составляет отличную точку контакта для поражения электрическим током) или полного контакта с большим металлическим предметом, таким как труба или металл рукоятки инструмента сопротивление корпуса может упасть до 1000 Ом (1 кОм), что приведет к тому, что даже более низкое напряжение может представлять потенциальную опасность.

    [латекс] E = IR [/ латекс]

    [латекс] E = (20 мА) (1 кОмега) [/ латекс]

    [латекс] \ textbf {E = 20 V} [/ латекс]

    Обратите внимание, что в этом состоянии 20 вольт достаточно, чтобы произвести ток в 20 миллиампер через человека; достаточно, чтобы вызвать столбняк. Помните, было высказано предположение, что сила тока всего 17 миллиампер может вызвать фибрилляцию желудочков (сердца). При сопротивлении рукопашной в 1000 Ом для создания этого опасного состояния потребуется всего 17 вольт.

    [латекс] E = IR [/ латекс]
    [латекс] E = (17 мА) (1 кВт) [/ латекс]
    [латекс] \ textbf {E = 17 В} [/ латекс]

    Семнадцать вольт — это не очень много для электрических систем. Конечно, это «наихудший» сценарий с напряжением переменного тока 60 Гц и отличной проводимостью тела, но он действительно показывает, насколько низкое напряжение может представлять серьезную угрозу при определенных условиях.

    Условия, необходимые для создания сопротивления тела 1000 Ом, не должны быть такими экстремальными, как то, что было представлено (потная кожа при контакте с золотым кольцом).Сопротивление тела может уменьшаться при приложении напряжения (особенно если столбняк заставляет пострадавшего крепче держать проводник), так что при постоянном напряжении удар может усилиться после первого контакта. То, что начинается как легкий шок — ровно настолько, чтобы «заморозить» жертву, чтобы она не могла отпустить ее, может перерасти в нечто достаточно серьезное, чтобы убить ее, поскольку сопротивление их тела уменьшается, а сила тока соответственно увеличивается.

    Research предоставило примерный набор цифр для электрического сопротивления точек контакта человека в различных условиях:

    Ситуация Сухой мокрый
    Проволока касалась пальцем 40000 Ом — 1000000 Ом 4000 Ом — 15000 Ом
    Проволока в руке 15000 Ом — 50 000 Ом 3000 Ом — 5000 Ом
    Ручные плоскогубцы по металлу 5000 Ом — 10 000 Ом 1000 Ом — 3000 Ом
    Контакт ладонью 3000 Ом — 8000 Ом 1000 Ом — 2000 Ом
    1.5-дюймовая металлическая труба с захватом одной рукой 1000 Ом — 3000 Ом 500 Ом — 1500 Ом
    1,5-дюймовая металлическая труба с захватом двумя руками 500 Ом — 1500 кОм 250 Ом — 750 Ом
    Рука погружена в проводящую жидкость 200 Ом — 500 Ом
    Опора, погруженная в проводящую жидкость 100 Ом — 300 Ом

    Обратите внимание на значения сопротивления для двух состояний с 1.5-дюймовая металлическая труба. Сопротивление, измеренное при захвате трубы двумя руками, составляет ровно половину сопротивления при захвате трубы одной рукой.

    Рис. 1.8

    Площадь соприкосновения с телом двумя руками в два раза больше, чем с одной рукой. Это важный урок: электрическое сопротивление между любыми контактирующими объектами уменьшается с увеличением площади контакта при прочих равных условиях. Если держать трубу двумя руками, ток имеет два параллельных маршрутов, по которым он течет от трубы к телу (или наоборот).

    Рисунок 1.9.

    Как мы увидим в более поздней главе, параллельных цепей всегда приводят к меньшему общему сопротивлению, чем любой отдельный путь, рассматриваемый отдельно.

    В промышленности 30 вольт обычно считается консервативным пороговым значением для опасного напряжения. Осторожный человек должен расценивать любое напряжение выше 30 В как опасное, не полагаясь на нормальное сопротивление тела для защиты от поражения электрическим током. Тем не менее, при работе с электричеством все же отличной идеей является держать руки чистыми и сухими и снимать все металлические украшения.Даже при более низком напряжении металлические украшения могут представлять опасность, поскольку проводят ток, достаточный для ожога кожи, при контакте между двумя точками в цепи. Металлические кольца, в частности, были причиной более чем нескольких ожогов пальцев из-за замыкания между точками в низковольтной и сильноточной цепи.

    Кроме того, напряжение ниже 30 может быть опасным, если его достаточно, чтобы вызвать неприятное ощущение, которое может вызвать вздрагивание и случайное соприкосновение с более высоким напряжением или другой опасностью.Я вспоминаю, как однажды жарким летним днем ​​работал над автомобилем. На мне были шорты, моя голая нога касалась хромового бампера автомобиля, когда я затягивал контакты аккумулятора. Когда я прикоснулся металлическим ключом к положительной (незаземленной) стороне 12-вольтовой батареи, я почувствовал покалывание в том месте, где моя нога касалась бампера. Сочетание плотного контакта с металлом и моей вспотевшей кожи позволило почувствовать шок всего лишь при напряжении 12 вольт.

    К счастью, ничего плохого не произошло, но если бы двигатель работал и удар ощущался в моей руке, а не ноге, я мог бы рефлекторно толкнуть руку на пути вращающегося вентилятора или уронить металлический ключ на клеммы аккумулятора (производя большое количество тока через гаечный ключ с большим количеством сопутствующих искр).Это иллюстрирует еще один важный урок, касающийся электробезопасности; этот электрический ток сам по себе может быть косвенной причиной травмы, заставляя вас подпрыгивать или спазмировать части вашего тела в опасную для вас сторону.

    Ток, проходящий через человеческое тело, имеет значение, насколько он опасен. Ток будет влиять на все мышцы, находящиеся на его пути, а поскольку мышцы сердца и легких (диафрагмы), вероятно, являются наиболее важными для выживания, пути удара, проходящие через грудную клетку, являются наиболее опасными.Это делает путь электрического тока из рук в руки очень вероятным способом получения травм и летального исхода.

    Во избежание подобных ситуаций рекомендуется работать с цепями под напряжением, находящимися под напряжением, только одной рукой, а вторую руку держать в кармане, чтобы случайно ни к чему не прикоснуться. Конечно, всегда безопаснее работать в цепи, когда она отключена, но это не всегда практично или возможно. При работе одной рукой, как правило, предпочтение отдается правой руке по двум причинам: большинство людей правши (что обеспечивает дополнительную координацию при работе), а сердце обычно находится слева от центра в грудной полости.

    Для левшей этот совет может быть не лучшим. Если такой человек недостаточно скоординирован с правой рукой, он может подвергнуть себя большей опасности, используя ту руку, с которой ему меньше всего комфортно, даже если электрический ток, протекающий через эту руку, может представлять большую опасность для его сердца. Относительная опасность между сотрясением одной рукой или другой, вероятно, меньше, чем опасность работы с менее чем оптимальной координацией, поэтому выбор руки для работы лучше всего оставить на усмотрение человека.

    Лучшая защита от ударов цепи под напряжением — это сопротивление, а сопротивление может быть добавлено к телу с помощью изолированных инструментов, перчаток, обуви и другого снаряжения. Ток в цепи является функцией доступного напряжения, деленного на общее сопротивление на пути потока. Как мы рассмотрим более подробно позже в этой книге, сопротивления имеют аддитивный эффект, когда они сложены так, что ток может течь только по одному пути:

    Рисунок 1.10

    Человек, находящийся в прямом контакте с источником напряжения: ток ограничен только сопротивлением тела.

    [латекс] I = \ frac {E} {R_ {boot}} [/ латекс]

    Теперь мы рассмотрим эквивалентную схему для человека в изолированных перчатках и ботинках:

    Рисунок 1.11

    Лицо в изоляционных перчатках и сапогах;

    Ток теперь ограничен сопротивлением цепи:

    [латекс] I = \ frac {E} {R_ {glove} + R_ {body} + R_ {boot} +} [/ latex]

    Поскольку электрический ток должен проходить через ботинок и , тело и перчатку, чтобы замкнуть цепь обратно к батарее, общая сумма ( сумма ) этих сопротивлений противодействует протеканию тока в большей степени, чем любое другое. сопротивлений рассматривается индивидуально.

    Безопасность — одна из причин, по которой электрические провода обычно покрывают пластиковой или резиновой изоляцией: чтобы значительно увеличить сопротивление между проводником и тем, кто или что-либо может с ним контактировать. К сожалению, было бы непомерно дорого изолировать проводники линии электропередач из-за недостаточной изоляции для обеспечения безопасности в случае случайного контакта. Таким образом, безопасность обеспечивается за счет того, что эти стропы должны находиться достаточно далеко вне досягаемости, чтобы никто не мог случайно их коснуться.

    Если возможно, отключите питание цепи перед выполнением каких-либо работ с ней.Вы должны обезопасить все источники вредной энергии, прежде чем систему можно будет считать безопасной для работы. В промышленности обеспечение безопасности цепи, устройства или системы в этом состоянии обычно называют переводом их в состояние нулевой энергии . В центре внимания этого урока, конечно же, электробезопасность. Однако многие из этих принципов применимы и к неэлектрическим системам.

    • Вред для тела зависит от силы электрического тока. Более высокое напряжение позволяет производить более высокие и опасные токи.Сопротивление противостоит току, поэтому высокое сопротивление является хорошей защитой от ударов.
    • Обычно считается, что любое напряжение выше 30 может создавать опасные ударные токи. Металлические украшения определенно плохо носить при работе с электрическими цепями. Кольца, ремешки для часов, ожерелья, браслеты и другие подобные украшения обеспечивают отличный электрический контакт с вашим телом и сами могут проводить ток, достаточный для возникновения ожогов кожи даже при низком напряжении.
    • Низкое напряжение может быть опасным, даже если оно слишком низкое, чтобы напрямую вызвать поражение электрическим током.Их может быть достаточно, чтобы напугать жертву, заставив ее отпрянуть и коснуться чего-то более опасного в непосредственной близости.
    • Когда необходимо работать с «живым» контуром, лучше всего выполнять работу одной рукой, чтобы предотвратить смертельный путь электрического тока из рук в руки (через грудную клетку).
    • Если возможно, отключите питание цепи перед выполнением каких-либо работ с ней.

    При работе с оборудованием отключите все источники питания перед выполнением любых работ.В промышленности удаление этих источников питания из схемы, устройства или системы обычно называется переводом в состояние с нулевым энергопотреблением . В центре внимания этого урока, конечно же, электробезопасность. Однако многие из этих принципов применимы и к неэлектрическим системам.

    Обеспечение безопасности чего-либо в состоянии нулевой энергии означает избавление от любого вида потенциальной или накопленной энергии, включая, помимо прочего:

    • Опасное напряжение
    • Давление пружины
    • Гидравлическое давление (жидкость)
    • Пневматическое (воздушное) давление
    • Подвес
    • Химическая энергия (легковоспламеняющиеся или иным образом реагирующие вещества)
    • Ядерная энергия (радиоактивные или делящиеся вещества)

    Напряжение по своей природе является проявлением потенциальной энергии.В первой главе я даже использовал приподнятую жидкость в качестве аналогии с потенциальной энергией напряжения, имеющей способность (потенциал) производить ток (поток), но не обязательно осознавая этот потенциал, пока не будет установлен подходящий путь для потока. и сопротивление потоку преодолевается. Пара проводов с высоким напряжением между ними не выглядит и не кажется опасной, даже если между ними имеется достаточно потенциальной энергии, чтобы протолкнуть смертоносное количество тока через ваше тело. Несмотря на то, что это напряжение в настоящее время ничего не делает, у него есть потенциал, и этот потенциал необходимо нейтрализовать, прежде чем можно будет физически контактировать с этими проводами.

    Все правильно спроектированные схемы имеют механизмы отключения для снятия напряжения в цепи. Иногда эти «разъединения» служат двойной цели: автоматически размыкаются в условиях чрезмерного тока, и в этом случае мы называем их «автоматическими выключателями». В других случаях выключатели-разъединители представляют собой устройства с ручным управлением без автоматической функции. В любом случае они существуют для вашей защиты и должны использоваться должным образом. Обратите внимание, что устройство отключения должно быть отдельно от обычного выключателя, используемого для включения и выключения устройства.Это предохранительный выключатель, который должен использоваться только для защиты системы в состоянии нулевого потребления энергии:

    Рисунок 1.12

    Когда разъединитель находится в «разомкнутом» положении, как показано (нет непрерывности), цепь разомкнута, и ток не будет существовать. На нагрузке будет нулевое напряжение, а полное напряжение источника будет падать на разомкнутые контакты выключателя. Обратите внимание, что в нижнем проводе цепи нет необходимости в размыкающем выключателе. Поскольку эта сторона цепи надежно соединена с землей (землей), она электрически является общей с землей, и ее лучше оставить таким образом.Для максимальной безопасности персонала, работающего с нагрузкой этой цепи, можно установить временное заземление на верхней стороне нагрузки, чтобы исключить падение напряжения на нагрузке:

    Рисунок 1.13

    При наличии временного заземляющего соединения обе стороны проводки нагрузки соединяются с землей, обеспечивая нулевое состояние энергии на нагрузке.

    Поскольку заземление с обеих сторон нагрузки электрически эквивалентно короткому замыканию через нагрузку с помощью провода, это еще один способ достижения той же цели максимальной безопасности:

    Рисунок 1.14

    В любом случае обе стороны нагрузки будут электрически общими с землей, с учетом отсутствия напряжения (потенциальной энергии) между обеими сторонами нагрузки и землей, на которой стоят люди. Этот метод временного заземления проводов в обесточенной энергосистеме очень распространен при работах по техническому обслуживанию, выполняемых в системах распределения электроэнергии высокого напряжения.

    Еще одним преимуществом этой меры предосторожности является защита от возможности включения размыкающего переключателя (включения его для обеспечения непрерывности цепи), когда люди все еще контактируют с нагрузкой.Временный провод, подключенный к нагрузке, создавал бы короткое замыкание, когда выключатель был замкнут, немедленно отключая любые устройства защиты от перегрузки по току (автоматические выключатели или предохранители) в цепи, что снова отключает питание. Если это произойдет, разъединитель вполне может получить повреждение, но рабочие на нагрузке находятся в безопасности.

    Здесь было бы хорошо упомянуть, что устройства максимального тока не предназначены для защиты от поражения электрическим током.Скорее, они существуют исключительно для защиты проводников от перегрева из-за чрезмерных токов. Только что описанные временные закорачивающие провода действительно могут вызвать «срабатывание» любых устройств перегрузки по току в цепи, если выключатель должен быть замкнут, но следует понимать, что защита от поражения электрическим током не является предполагаемой функцией этих устройств. Их основная функция будет просто использоваться для защиты рабочего с установленным закорачивающим проводом.

    Структурированные системы безопасности: блокировка / маркировка

    Поскольку очевидно, что важно иметь возможность закрепить любые отключающие устройства в разомкнутом (выключенном) положении и убедиться, что они остаются в этом положении во время работы в цепи, существует потребность в структурированной системе безопасности, которая должна быть введена в место.Такая система обычно используется в промышленности и называется Lock-out / Tag-out .

    Процедура блокировки / маркировки работает следующим образом: все люди, работающие в защищенной цепи, имеют свой собственный замок или кодовый замок, который они устанавливают на рычаге управления устройства отключения перед работой с системой. Кроме того, они должны заполнить и подписать ярлык, который они вешают на свой замок, с описанием характера и продолжительности работы, которую они собираются выполнять в системе.Если есть несколько источников энергии, которые необходимо «заблокировать» (множественные разъединения, как электрические, так и механические источники энергии должны быть защищены, и т. Д.), Рабочий должен использовать столько своих замков, сколько необходимо для обеспечения питания от системы. до начала работы. Таким образом, система поддерживается в состоянии нулевого энергопотребления до тех пор, пока не будет снята каждая последняя блокировка со всех устройств отключения и отключения, а это означает, что каждый последний работник даст согласие, сняв свои личные блокировки. Если будет принято решение повторно активировать систему, и замок (и) одного человека все еще остается на месте после того, как все присутствующие снимают свои, метка (и) покажет, кто этот человек и что он делает.

    Даже при наличии хорошей программы обеспечения безопасности по блокировке / маркировке все равно необходимо проявлять усердие и разумные меры предосторожности. Это особенно актуально в промышленных условиях, когда над устройством или системой может одновременно работать множество людей. Некоторые из этих людей могут не знать о надлежащей процедуре блокировки / маркировки или могут знать о ней, но слишком самоуверенны, чтобы ей следовать. Не думайте, что все соблюдают правила безопасности!

    После того, как электрическая система была заблокирована и помечена вашим личным замком, вы должны дважды проверить, действительно ли напряжение зафиксировано в нулевом состоянии.Один из способов проверить — увидеть, запустится ли машина (или что-то еще, над чем она работает), если будет задействован переключатель или кнопка start . Если он запускается, значит, вы знаете, что не смогли получить от него электроэнергию.

    Кроме того, вы должны всегда проверять на наличие опасного напряжения с помощью измерительного прибора, прежде чем касаться каких-либо проводов в цепи. Для большей безопасности вы должны выполнить следующую процедуру проверки, использования, а затем проверки вашего глюкометра:

    • Убедитесь, что ваш измеритель правильно показывает на известном источнике напряжения.
    • Используйте свой измеритель, чтобы проверить цепь блокировки на наличие опасного напряжения.
    • Еще раз проверьте свой измеритель на известном источнике напряжения, чтобы убедиться, что он по-прежнему показывает должным образом.

    Хотя это может показаться чрезмерным или даже параноидальным, это проверенный метод предотвращения поражения электрическим током. Однажды у меня был счетчик, который не смог показать напряжение, когда он должен был, при проверке цепи, чтобы убедиться, что она «мертвая». Если бы я не использовал другие средства для проверки наличия напряжения, меня бы сегодня не было в живых, чтобы написать это.Всегда есть шанс, что ваш вольтметр окажется неисправным именно тогда, когда он понадобится вам для проверки на наличие опасного состояния. Следуя этим инструкциям, вы никогда не попадете в смертельную ситуацию из-за поломки счетчика.

    Наконец, электротехник прибудет к тому моменту процедуры проверки безопасности, когда будет считаться безопасным прикосновение к проводнику (проводам). Имейте в виду, что после принятия всех мер предосторожности возможно (хотя и очень маловероятно) наличие опасного напряжения.Последней мерой предосторожности, которую следует предпринять на этом этапе, является кратковременный контакт проводника (проводов) тыльной стороной руки перед тем, как схватить его или металлический инструмент, соприкасающийся с ним. Почему? Если по какой-то причине между этим проводником и заземлением все еще присутствует напряжение, движение пальца в результате реакции удара (сжатие в кулак) приведет к разрыву контакта с проводником. Обратите внимание, что это абсолютно последний шаг , который должен выполнить любой электромонтер перед началом работы с энергосистемой, и никогда не следует использовать в качестве альтернативного метода проверки опасного напряжения.Если у вас когда-либо будут основания сомневаться в надежности вашего глюкометра, воспользуйтесь другим глюкометром, чтобы получить «второе мнение».

    • Состояние нулевой энергии: Когда цепь, устройство или система защищены таким образом, что не существует потенциальной энергии, которая могла бы нанести вред кому-либо, работающему с ними.
    • Выключатели-разъединители
    • должны присутствовать в правильно спроектированной электрической системе, чтобы обеспечить удобную готовность к состоянию нулевого потребления энергии.
    • К обслуживаемой нагрузке могут быть подключены временные заземляющие или закорачивающие провода для дополнительной защиты персонала, работающего с этой нагрузкой.
    • Блокировка / маркировка
    • работает следующим образом: при работе с системой в состоянии нулевого энергопотребления рабочий помещает личный замок или кодовый замок на каждое устройство отключения энергии, имеющее отношение к его или ее задаче в этой системе. Кроме того, на каждый из этих замков навешивается тег, описывающий характер и продолжительность работы, которую необходимо выполнить, и того, кто ее выполняет.
    • Всегда проверяйте, что цепь была зафиксирована в состоянии нулевого потребления энергии с помощью испытательного оборудования после «блокировки». Обязательно проверьте свой глюкометр до и после проверки цепи, чтобы убедиться, что она работает правильно.
    • Когда придет время реально коснуться проводника (проводов) предположительно неработающей энергосистемы, сначала сделайте это тыльной стороной руки, чтобы в случае удара током реакция мышц оттолкнула пальцы от проводника. .

    Безопасное и эффективное использование электрического счетчика — это, пожалуй, самый ценный навык, которым может овладеть электронщик, как ради собственной безопасности, так и для профессионального мастерства. Поначалу может быть сложно использовать счетчик, зная, что вы подключаете его к цепям под напряжением, которые могут содержать опасные для жизни уровни напряжения и тока.Это опасение небезосновательно, и всегда лучше действовать осторожно при использовании счетчиков. Небрежность больше, чем какой-либо другой фактор, является причиной несчастных случаев с электричеством у опытных технических специалистов.

    Мультиметры

    Самым распространенным электрическим испытательным оборудованием является мультиметр . Мультиметры названы так потому, что они могут измерять множество переменных: напряжение, ток, сопротивление и часто многие другие, некоторые из которых не могут быть объяснены здесь из-за их сложности.В руках обученного техника мультиметр является одновременно эффективным рабочим инструментом и защитным устройством. Однако в руках невежественного и / или неосторожного человека мультиметр может стать источником опасности при подключении к «действующей» цепи.

    Существует много разных марок мультиметров, причем каждый производитель выпускает несколько моделей с разными наборами функций. Мультиметр, показанный здесь на следующих иллюстрациях, представляет собой «общую» конструкцию, не специфичную для какого-либо производителя, но достаточно общую, чтобы научить основным принципам использования:

    Рисунок 1.15

    Вы заметите, что дисплей этого измерителя имеет «цифровой» тип: числовые значения отображаются с использованием четырех цифр, как на цифровых часах. Поворотный селекторный переключатель (теперь установлен в положение Off ) имеет пять различных положений измерения, в которых он может быть установлен: два значения «V», два значения «A» и одно положение посередине с забавной «подковой». Символ на нем, представляющий «сопротивление». Символ «подкова» — это греческая буква «Омега» (Ω), которая является общим символом для электрической единицы измерения ом.

    Из двух настроек «V» и двух настроек «A» вы заметите, что каждая пара разделена на уникальные маркеры либо парой горизонтальных линий (одна сплошная, одна пунктирная), либо пунктирной линией с волнистой кривой над ней. . Параллельные линии представляют «постоянный ток», а волнистая кривая — «переменный ток». «V», конечно, означает «напряжение», а «A» означает «сила тока» (ток). Измеритель использует другие методы для измерения постоянного тока внутри, чем он использует для измерения переменного тока, и поэтому он требует от пользователя выбора типа напряжения (В) или тока (А) для измерения.Хотя мы не обсуждали переменный ток (AC) в каких-либо технических деталях, это различие в настройках счетчика важно помнить.

    Мультиметр Розетки

    На лицевой панели мультиметра есть три разных гнезда, к которым мы можем подключить наши измерительные провода . Измерительные провода — это не что иное, как специально подготовленные провода, используемые для подключения измерителя к тестируемой цепи. Провода покрыты гибкой изоляцией с цветовой кодировкой (черной или красной), чтобы руки пользователя не касались оголенных проводов, а концы зондов представляют собой острые жесткие кусочки проволоки:

    Рисунок 1.16

    Черный измерительный провод всегда подключается к черному разъему на мультиметре: с пометкой «COM» для «общего». Красные измерительные провода подключаются либо к красному разъему с маркировкой напряжения и сопротивления, либо к красному разъему с маркировкой тока, в зависимости от того, какое количество вы собираетесь измерить с помощью мультиметра.

    Чтобы увидеть, как это работает, давайте посмотрим на пару примеров, показывающих, как используется счетчик. Сначала мы настроим измеритель для измерения постоянного напряжения от батареи:

    Рисунок 1.17

    Обратите внимание, что два измерительных провода подключены к соответствующим гнездам на измерителе для измерения напряжения, а селекторный переключатель установлен на «V» постоянного тока. Теперь рассмотрим пример использования мультиметра для измерения напряжения переменного тока от бытовой электрической розетки (настенной розетки):

    Рисунок 1.18

    Единственное отличие в настройке счетчика — это расположение селекторного переключателя: теперь он установлен на переменный ток «V». Поскольку мы все еще измеряем напряжение, измерительные провода останутся подключенными к тем же гнездам.В обоих этих примерах настоятельно требует, чтобы не позволяли наконечникам щупов соприкасаться друг с другом, пока они оба находятся в контакте со своими соответствующими точками в цепи. Если это произойдет, образуется короткое замыкание, вызывающее искру и, возможно, даже шар пламени, если источник напряжения способен обеспечить достаточный ток! Следующее изображение иллюстрирует потенциальную опасность:

    Рис. 1.19.

    Это лишь один из способов, которым счетчик может стать источником опасности при неправильном использовании.

    Измерение напряжения, пожалуй, самая распространенная функция, для которой используется мультиметр. Это, безусловно, первичное измерение, выполняемое в целях безопасности (часть процедуры блокировки / маркировки), и оно должно быть хорошо понято оператором счетчика. Поскольку напряжение между двумя точками всегда относительное, измеритель должен быть надежно подключен к двум точкам в цепи, прежде чем он будет обеспечивать надежное измерение. Обычно это означает, что оба щупа должны быть схвачены руками пользователя и прижаты к правильным точкам контакта источника напряжения или цепи во время измерения.

    Поскольку путь электрического тока из рук в руки является наиболее опасным, удерживание измерительных щупов в двух точках высоковольтной цепи таким образом всегда представляет потенциальную опасность . Если защитная изоляция на датчиках изношена или потрескалась, пальцы пользователя могут соприкоснуться с проводниками датчика во время испытания, что приведет к сильному удару. Это более безопасный вариант, если можно использовать только одну руку для захвата зондов. Иногда можно «защелкнуть» один наконечник щупа на контрольной точке цепи, чтобы его можно было отпустить, а другой установить на место, используя только одну руку.Для облегчения этого можно прикрепить специальные аксессуары для наконечников зонда, такие как пружинные зажимы.

    Помните, что измерительные провода измерителя являются частью всего комплекта оборудования и что с ними следует обращаться так же осторожно и уважительно, как и с самим измерителем. Если вам нужен специальный аксессуар для ваших измерительных проводов, такой как пружинный зажим или другой специальный наконечник зонда, обратитесь к каталогу продукции производителя измерителя или другого производителя испытательного оборудования. Не пытайтесь проявить изобретательность и делать свои собственные пробники , так как вы можете подвергнуть себя опасности в следующий раз, когда будете использовать их в цепи под напряжением.

    Также следует помнить, что цифровые мультиметры обычно хорошо справляются с различением измерений переменного и постоянного тока, поскольку они настраиваются на одно или другое при проверке напряжения или тока. Как мы видели ранее, как переменное, так и постоянное напряжение и ток могут быть смертельными, поэтому при использовании мультиметра в качестве устройства проверки безопасности вы всегда должны проверять наличие как переменного, так и постоянного тока, даже если вы не ожидаете найти и то, и другое. ! Кроме того, при проверке наличия опасного напряжения вы должны обязательно проверить всех пар рассматриваемых точек.

    Например, предположим, что вы открыли шкаф с электропроводкой и обнаружили три больших проводника, подающих питание переменного тока на нагрузку. Автоматический выключатель, питающий эти провода (предположительно), был отключен, заблокирован и помечен. Вы дважды проверили отсутствие питания, нажав кнопку Start для нагрузки. Ничего не произошло, поэтому теперь вы переходите к третьему этапу проверки безопасности: проверке измерителя напряжения.

    Сначала вы проверяете свой измеритель на известном источнике напряжения, чтобы убедиться, что он работает правильно.Любая ближайшая электрическая розетка должна обеспечивать удобный источник переменного напряжения для проверки. Вы делаете это и обнаруживаете, что счетчик показывает как следует. Затем вам нужно проверить напряжение между этими тремя проводами в шкафу. Но напряжение измеряется между и двумя точками , так где же проверить?

    Рисунок 1.20

    Ответ — проверить все комбинации этих трех точек. Как видите, на рисунке точки обозначены буквами «A», «B» и «C», поэтому вам нужно будет взять мультиметр (установленный в режиме вольтметра) и проверить его между точками A и B, B и C, а также A и C.Если вы обнаружите напряжение между любой из этих пар, цепь не находится в состоянии нулевой энергии. Но ждать! Помните, что мультиметр не будет регистрировать напряжение постоянного тока, когда он находится в режиме переменного напряжения, и наоборот, поэтому вам необходимо проверить эти три пары точек в в каждом режиме , в общей сложности шесть проверок напряжения для завершения!

    Однако, даже несмотря на всю эту проверку, мы еще не охватили все возможности. Помните, что опасное напряжение может появиться между одиночным проводом и землей (в этом случае металлический каркас шкафа будет хорошей точкой отсчета заземления) в энергосистеме.Итак, чтобы быть в полной безопасности, мы должны не только проверять между A и B, B и C, и A и C (как в режимах переменного, так и постоянного тока), но мы также должны проверять между A и землей, B и землей, и C & заземление (как в режимах переменного, так и постоянного тока)! Это дает в общей сложности двенадцать проверок напряжения для этого, казалось бы, простого сценария всего с тремя проводами. Затем, конечно же, после того, как мы завершили все эти проверки, нам нужно взять мультиметр и повторно проверить его с помощью известного источника напряжения, такого как розетка, чтобы убедиться, что он по-прежнему в хорошем рабочем состоянии.

    Использование мультиметра для проверки сопротивления

    Использование мультиметра для проверки сопротивления — гораздо более простая задача. Измерительные провода будут оставаться подключенными к тем же розеткам, что и для проверки напряжения, но селекторный переключатель необходимо повернуть, пока он не укажет на символ сопротивления «подкова». Касаясь щупами устройства, сопротивление которого необходимо измерить, прибор должен правильно отображать сопротивление в омах:

    Рисунок 1.21

    При измерении сопротивления следует помнить, что это нужно делать только на обесточенных компонентах ! Когда измеритель находится в режиме «сопротивления», он использует небольшую внутреннюю батарею для генерации крошечного тока через измеряемый компонент. Путем определения того, насколько сложно пропустить этот ток через компонент, можно определить и отобразить сопротивление этого компонента. Если в контуре измерителя-вывод-компонент-вывод-измеритель имеется дополнительный источник напряжения, который либо помогает, либо противодействует току измерения сопротивления, производимому измерителем, это приведет к ошибочным показаниям.В худшем случае счетчик может даже выйти из строя из-за внешнего напряжения.

    Режим «Сопротивление» мультиметра

    Режим «сопротивления» мультиметра очень полезен для определения целостности проводов, а также для точных измерений сопротивления. Когда между наконечниками пробников имеется хорошее, прочное соединение (моделируется путем их соприкосновения), измеритель показывает почти нулевое сопротивление. Если бы измерительные провода не имели сопротивления, он показывал бы ровно ноль:

    . Рисунок 1.22

    Если выводы не соприкасаются друг с другом или не касаются противоположных концов разорванного провода, измеритель покажет бесконечное сопротивление (обычно путем отображения пунктирных линий или аббревиатуры «O.L.», что означает «разомкнутый контур»):

    Рисунок 1.23

    Измерение тока с помощью мультиметра

    Безусловно, наиболее опасным и сложным применением мультиметра является измерение тока. Причина этого довольно проста: для того, чтобы измеритель мог измерять ток, измеряемый ток должен проходить через счетчика.Это означает, что измеритель должен быть частью цепи тока, а не просто подключаться к какой-либо стороне, как в случае измерения напряжения. Чтобы сделать измеритель частью пути тока цепи, исходная цепь должна быть «разорвана», а измеритель должен быть подключен к двум точкам разомкнутого разрыва. Чтобы настроить измеритель на это, селекторный переключатель должен указывать на переменный или постоянный ток «A», а красный измерительный провод должен быть вставлен в красную розетку с маркировкой «A». На следующем рисунке показан измеритель, полностью готовый к измерению тока, и проверяемая цепь:

    Рисунок 1.24

    Сейчас цепь разомкнута при подготовке к подключению счетчика:

    Рисунок 1.25

    Следующий шаг — вставить измеритель в линию со схемой, подключив два наконечника щупа к разомкнутым концам цепи, черный щуп к отрицательной (-) клемме 9-вольтовой батареи и красный щуп к свободному концу провода, ведущему к лампе:

    Рисунок 1.26

    Этот пример показывает очень безопасную схему для работы. 9 вольт вряд ли представляют опасность поражения электрическим током, поэтому не стоит бояться разомкнуть эту цепь (не голыми руками, не меньше!) И подключить счетчик параллельно с током.Однако с цепями более высокой мощности это действительно может быть опасным занятием. Даже если напряжение в цепи было низким, нормальный ток мог быть достаточно высоким, чтобы возникла опасная искра в момент установления последнего подключения датчика измерителя.

    Другой потенциальной опасностью использования мультиметра в режиме измерения тока («амперметр») является невозможность правильно вернуть его в конфигурацию измерения напряжения перед измерением напряжения с его помощью. Причины этого зависят от конструкции и работы амперметра.При измерении тока в цепи путем размещения измерителя непосредственно на пути тока, лучше всего, чтобы измеритель оказывал небольшое сопротивление току или не оказывал никакого сопротивления. В противном случае дополнительное сопротивление изменит работу схемы. Таким образом, мультиметр спроектирован так, чтобы сопротивление между наконечниками измерительного щупа было практически нулевым, когда красный щуп был вставлен в красное гнездо «А» (для измерения тока). В режиме измерения напряжения (красный провод вставлен в красное гнездо «V») между наконечниками измерительных щупов имеется большое количество мегаомов сопротивления, потому что вольтметры имеют сопротивление, близкое к бесконечному (так что они не работают). t потребляет значительный ток от тестируемой цепи).

    При переключении мультиметра из режима измерения тока в режим измерения напряжения легко повернуть селекторный переключатель из положения «A» в положение «V» и забыть, соответственно, переключить положение разъема красного измерительного провода с «A» на положение «V». «V». В результате — если счетчик затем подключить к источнику значительного напряжения — произойдет короткое замыкание счетчика!

    Рисунок 1.27.

    Чтобы предотвратить это, большинство мультиметров имеют функцию предупреждения, с помощью которой они издают звуковой сигнал, если когда-либо в гнездо «A» вставлен провод, а селекторный переключатель установлен в положение «V».Однако какими бы удобными ни были эти функции, они по-прежнему не заменяют ясного мышления и осторожности при использовании мультиметра.

    Все качественные мультиметры содержат внутри предохранители, которые спроектированы так, чтобы «перегорать» в случае чрезмерного тока через них, как в случае, показанном на последнем изображении. Как и все устройства максимальной токовой защиты, эти предохранители в первую очередь предназначены для защиты оборудования (в данном случае самого счетчика) от чрезмерных повреждений и только во вторую очередь для защиты пользователя от повреждений.Мультиметр можно использовать для проверки собственного предохранителя, установив селекторный переключатель в положение сопротивления и создав соединение между двумя красными гнездами следующим образом:

    Рисунок 1.28.

    . Исправный предохранитель будет указывать на очень низкое сопротивление, в то время как перегоревший предохранитель всегда будет показывать «O.L.» (или любое другое указание, которое используется в этой модели мультиметра для обозначения отсутствия непрерывности). Фактическое количество Ом, отображаемое для исправного предохранителя, не имеет большого значения, если оно является произвольно низким.

    Итак, теперь, когда мы увидели, как использовать мультиметр для измерения напряжения, сопротивления и тока, что еще нужно знать? Множество! Ценность и возможности этого универсального испытательного прибора станут более очевидными по мере того, как вы приобретете навыки и познакомитесь с ним.Ничто не заменит регулярных занятий со сложными инструментами, такими как эти, поэтому не стесняйтесь экспериментировать с безопасными схемами с батарейным питанием.

    • Измеритель, способный проверять напряжение, ток и сопротивление, называется мультиметром .
    • Поскольку напряжение между двумя точками всегда относительное, измеритель напряжения («вольтметр») должен быть подключен к двум точкам в цепи, чтобы получить хорошие показания. Будьте осторожны, не касайтесь оголенных наконечников щупов вместе при измерении напряжения, так как это приведет к короткому замыканию!
    • Не забывайте всегда проверять как напряжение переменного, так и постоянного тока при использовании мультиметра для проверки наличия опасного напряжения в цепи.Убедитесь, что вы проверяете напряжение между всеми комбинациями пар проводников, в том числе между отдельными проводниками и землей!
    • В режиме измерения напряжения («вольтметр») мультиметры имеют очень высокое сопротивление между выводами.
    • Никогда не пытайтесь измерить сопротивление или целостность цепи с помощью мультиметра в цепи, которая находится под напряжением. В лучшем случае показания сопротивления, полученные от глюкометра, будут неточными, а в худшем случае глюкометр может быть поврежден, а вы можете получить травму.
    • Измерители тока («амперметры») всегда подключены в цепь, поэтому электроны должны проходить через через счетчик.
    • В режиме измерения тока («амперметр») мультиметры практически не имеют сопротивления между выводами. Это сделано для того, чтобы электроны могли проходить через счетчик с наименьшими трудностями. Если бы это было не так, измеритель добавлял бы дополнительное сопротивление в цепи, тем самым влияя на ток.

    Как мы видели ранее, энергосистема без надежного соединения с землей непредсказуема с точки зрения безопасности.Невозможно гарантировать, какое или как мало будет напряжения между любой точкой цепи и землей. Заземлив одну сторону источника напряжения энергосистемы, по крайней мере, одна точка в цепи может быть электрически соединена с землей и, следовательно, не представляет опасности поражения электрическим током. В простой двухпроводной системе электропитания проводник, подключенный к земле, называется нейтраль , а другой провод называется горячий , также известный как live или active :

    . Рисунок 1.29 Двухпроводная система электропитания

    Что касается источника напряжения и нагрузки, заземление не имеет никакого значения. Он существует исключительно ради личной безопасности, гарантируя, что по крайней мере одна точка в цепи будет безопасна для прикосновения (нулевое напряжение относительно земли). «Горячая» сторона цепи, названная так из-за ее потенциальной опасности поражения электрическим током, будет опасна прикасаться, если напряжение не будет обеспечено путем надлежащего отключения от источника (в идеале, с использованием процедуры систематической блокировки / маркировки).

    Этот дисбаланс опасностей между двумя проводниками в простой силовой цепи важно понимать. Следующая серия иллюстраций основана на распространенных бытовых системах электропроводки (для простоты с использованием источников постоянного напряжения, а не переменного тока).

    Если мы посмотрим на простой бытовой электроприбор, такой как тостер с проводящим металлическим корпусом, мы увидим, что при правильной работе не должно быть опасности поражения электрическим током. Провода, передающие питание на нагревательные элементы тостера, изолированы от соприкосновения с металлическим корпусом (и друг с другом) резиной или пластиком.

    Рисунок 1.30 Отсутствие напряжения между корпусом и землей

    Однако, если один из проводов внутри тостера случайно войдет в контакт с металлическим корпусом, корпус станет электрически общим для провода, и прикосновение к корпусу будет столь же опасным, как прикосновение к оголенному проводу. Представляет ли это опасность поражения электрическим током, зависит от , к которому случайно прикоснется провод :

    Рисунок 1.31 случайное контактное напряжение между корпусом и землей

    Если «горячий» провод касается корпуса, это подвергает опасности пользователя тостера.С другой стороны, если нейтральный провод касается корпуса, опасности поражения электрическим током нет:

    Рисунок 1.32 Случайное отсутствие напряжения между корпусом и землей

    Чтобы гарантировать, что первый отказ менее вероятен, чем второй, инженеры стараются проектировать устройства таким образом, чтобы свести к минимуму контакт горячего проводника с корпусом. В идеале, конечно, вы не хотите, чтобы какой-либо из проводов случайно соприкасался с токопроводящим корпусом прибора, но обычно есть способы спроектировать расположение частей, чтобы сделать случайный контакт менее вероятным для одного провода, чем для другого.

    Однако эта профилактическая мера эффективна только в том случае, если может быть гарантирована полярность вилки питания. Если вилку можно перевернуть, то проводник с большей вероятностью соприкоснется с корпусом вполне может быть «горячим»:

    Рисунок 1.33 Напряжение между корпусом и землей

    Устройства, разработанные таким образом, обычно поставляются с «поляризованными» вилками, причем один контакт вилки немного уже, чем другой. Розетки питания также имеют такую ​​же конструкцию, причем один слот уже другой.Следовательно, вилку нельзя вставить «задом наперед», и можно гарантировать идентичность проводника внутри устройства. Помните, что это никак не влияет на основные функции устройства: это делается исключительно ради безопасности пользователя.

    Некоторые инженеры решают проблему безопасности, просто делая внешний корпус прибора непроводящим. Такие приборы называются с двойной изоляцией , поскольку изолирующий кожух служит вторым слоем изоляции над и за пределами самих проводов.Если провод внутри устройства случайно войдет в контакт с корпусом, это не представляет опасности для пользователя устройства.

    Другие инженеры решают проблему безопасности, поддерживая проводящий корпус, но используя третий провод для надежного соединения этого корпуса с землей:

    Рис. 1.34 Нулевое напряжение корпуса заземления между корпусом и землей

    Третий контакт на шнуре питания обеспечивает прямое электрическое соединение корпуса устройства с землей, делая две точки электрически общими друг с другом.Если они электрически общие, то между ними не может быть падения напряжения. По крайней мере, так оно и должно работать. Если горячий провод случайно коснется металлического корпуса прибора, он вызовет прямое короткое замыкание обратно на источник напряжения через заземляющий провод, сработав любые устройства защиты от сверхтока. Пользователь устройства останется в безопасности.

    Вот почему так важно никогда не отрезать третий контакт вилки питания, когда пытаетесь вставить его в розетку с двумя контактами.Если это будет сделано, не будет заземления корпуса прибора для обеспечения безопасности пользователя (ей). Устройство по-прежнему будет функционировать должным образом, но если возникнет внутренняя неисправность, в результате которой горячий провод соприкасается с корпусом, результаты могут быть смертельными. Если необходимо использовать двухконтактную розетку , можно установить двухконтактный переходник розетки с заземляющим проводом, прикрепленным к заземляющему винту крышки. Это обеспечит безопасность заземленного прибора, подключенного к розетке этого типа.

    Однако электрическая безопасность не обязательно заканчивается нагрузкой. Последнюю защиту от поражения электрическим током можно установить на стороне источника питания цепи, а не на самом приборе. Эта мера защиты называется , обнаружение замыкания на землю , и работает она так:

    В правильно функционирующем приборе (показанном выше) ток, измеренный через проводник под напряжением, должен быть точно равен току через нейтральный проводник, потому что существует только один путь для прохождения электронов в цепи.При отсутствии неисправности внутри устройства нет соединения между проводниками цепи и человеком, касающимся корпуса, и, следовательно, нет удара.

    Однако, если горячая проволока случайно коснется металлического корпуса, через человека, прикоснувшегося к корпусу, пройдет ток. Наличие ударного тока будет проявляться как разница в разности и тока между двумя силовыми проводниками в розетке:

    Рисунок 1.35 Разница в токе между двумя силовыми проводниками в розетке

    Эта разница в токе между «горячим» и «нейтральным» проводниками будет существовать только в том случае, если есть ток через заземление, что означает, что в системе есть неисправность.Следовательно, такая разница тока может использоваться как способ обнаружения состояния неисправности. Если устройство настроено для измерения этой разницы в токах между двумя силовыми проводниками, обнаружение дисбаланса тока может быть использовано для запуска размыкания выключателя, тем самым отключая питание и предотвращая серьезный удар:

    Рисунок 1.36 Прерыватели тока замыкания на землю

    Такие устройства называются прерывателями тока замыкания на землю , или сокращенно GFCI. За пределами Северной Америки GFCI известен как предохранительный выключатель, устройство защитного отключения (RCD), RCBO или RCD / MCB в сочетании с миниатюрным автоматическим выключателем или выключателем утечки на землю (ELCB).Они достаточно компактны, чтобы их можно было встроить в розетку. Эти розетки легко идентифицировать по их характерным кнопкам «Тест» и «Сброс». Большим преимуществом использования этого подхода для обеспечения безопасности является то, что он работает независимо от конструкции устройства. Конечно, использование прибора с двойной изоляцией или заземлением в дополнение к розетке GFCI было бы еще лучше, но приятно знать, что что-то может быть сделано для повышения безопасности помимо конструкции и состояния прибора.

    Прерыватель цепи дугового замыкания (AFCI) , автоматический выключатель, предназначенный для предотвращения пожаров, предназначен для размыкания при прерывистых резистивных коротких замыканиях. Например, нормальный выключатель на 15 А предназначен для быстрого размыкания цепи при нагрузке, значительно превышающей номинальную 15 А, или медленнее, немного превышающей номинальную. Хотя это защищает от прямого короткого замыкания и нескольких секунд перегрузки, соответственно, он не защищает от дуги — аналогично дуговой сварке. Дуга представляет собой сильно изменяющуюся нагрузку, периодически достигающую максимума более 70 А, разомкнутую цепь с переходами через ноль переменного тока.Хотя среднего тока недостаточно для срабатывания стандартного выключателя, его достаточно, чтобы разжечь пожар. Эта дуга может быть создана из-за металлического короткого замыкания, которое сжигает металл, оставляя резистивную распыляющую плазму ионизированных газов.

    AFCI содержит электронную схему для обнаружения этого прерывистого резистивного короткого замыкания. Он защищает как от дуги от горячего к нейтральному, так и от горячего к заземлению. AFCI не защищает от опасности поражения электрическим током, как GFCI. Таким образом, GFCI по-прежнему необходимо устанавливать на кухне, в ванной и на открытом воздухе.Поскольку AFCI часто срабатывает при запуске больших двигателей и, в более общем случае, щеточных двигателей, его установка ограничена электрическими цепями в спальнях в соответствии с Национальным электротехническим кодексом США. Использование AFCI должно уменьшить количество электрических пожаров. Однако неприятные срабатывания при работе приборов с двигателями в цепях AFCI представляют собой проблему.

    • В энергосистемах одна сторона источника напряжения часто подключается к заземлению для обеспечения безопасности в этой точке.
    • «Заземленный» провод в энергосистеме называется нулевым проводом , а незаземленный провод — горячим проводом .
    • Заземление в энергосистемах существует ради личной безопасности, а не для работы нагрузки (ей).
    • Электробезопасность прибора или других нагрузок может быть улучшена с помощью хорошей инженерии: поляризованные вилки, двойная изоляция и трехконтактные вилки с «заземлением» — все это способы повышения безопасности на стороне нагрузки.
    • Прерыватели тока замыкания на землю (GFCI) работают, считывая разницу в токе между двумя проводниками, подающими питание на нагрузку.Никакой разницы в токе быть не должно. Любая разница означает, что ток должен входить в нагрузку или выходить из нее каким-либо образом, кроме двух основных проводников, что нехорошо. Значительная разница в токе автоматически откроет размыкающий механизм выключателя, полностью отключив питание.

    Обычно допустимая токовая нагрузка проводника — это предел конструкции схемы, который нельзя намеренно превышать, но есть приложение, в котором ожидается превышение допустимой токовой нагрузки: в случае предохранителей .

    Что такое предохранитель?

    A Предохранитель представляет собой устройство электробезопасности, построенное вокруг проводящей полосы, которая предназначена для плавления и разделения в случае чрезмерного тока. Предохранители всегда подключаются последовательно с компонентами, которые должны быть защищены от перегрузки по току, поэтому, когда предохранитель перегорает (размыкается), он размыкает всю цепь и останавливает ток через компонент (ы). Плавкий предохранитель, включенный в одну ветвь параллельной цепи, конечно, не повлияет на ток через любую из других ветвей.

    Обычно тонкий кусок плавкой проволоки помещается в защитную оболочку, чтобы свести к минимуму опасность возникновения дугового разряда в случае прорыва проволоки с большой силой, что может произойти в случае сильных перегрузок по току. В случае небольших автомобильных предохранителей оболочка прозрачна, так что плавкий элемент может быть визуально осмотрен. В бытовой электропроводке обычно используются ввинчиваемые предохранители со стеклянным корпусом и тонкой узкой полосой из металлической фольги посередине. Фотография, на которой показаны оба типа предохранителей, представлена ​​здесь:

    Рисунок 1.37 Типы предохранителей

    Предохранители картриджного типа популярны в автомобильной промышленности и в промышленности, если они изготовлены из материалов оболочки, отличных от стекла. Поскольку предохранители рассчитаны на «отказ» срабатывания при превышении их номинального тока, они обычно предназначены для легкой замены в цепи. Это означает, что они будут вставлены в какой-либо тип держателя, а не припаиваться или прикрепляться болтами к проводникам цепи. Ниже приведена фотография, на которой изображена пара предохранителей со стеклянным картриджем в держателе с несколькими предохранителями:

    Рисунок 1.38 Стеклянный патрон с предохранителями Держатель нескольких предохранителей

    Предохранители удерживаются пружинными металлическими зажимами, причем сами зажимы постоянно соединены с проводниками цепи. Основной материал держателя предохранителя (или блока предохранителей , как их иногда называют) выбран как хороший изолятор.

    Другой тип держателя предохранителей патронного типа обычно используется для установки в панелях управления оборудованием, где желательно скрыть все точки электрического контакта от контакта с человеком.В отличие от только что показанного блока предохранителей, где все металлические зажимы открыты, этот тип держателя предохранителя полностью закрывает предохранитель в изолирующем корпусе:

    Рисунок 1.39 Патрон предохранителя закрывает изолирующий корпус

    Наиболее распространенным устройством для защиты от перегрузки по току в сильноточных цепях сегодня является автоматический выключатель .

    Что такое автоматический выключатель?

    Автоматические выключатели — это специально разработанные переключатели, которые автоматически размыкаются для отключения тока в случае перегрузки по току.Малые автоматические выключатели, такие как те, которые используются в жилых, коммерческих и легких промышленных предприятиях, имеют термическое управление. Они содержат биметаллическую полосу (тонкую полоску из двух металлов, соединенных спина к спине), несущую ток цепи, которая изгибается при нагревании. Когда биметаллическая полоса создает достаточную силу (из-за чрезмерного нагрева полосы), срабатывает механизм отключения, и прерыватель размыкается. Автоматические выключатели большего размера автоматически активируются силой магнитного поля, создаваемого токонесущими проводниками внутри выключателя, или могут срабатывать для отключения от внешних устройств, контролирующих ток цепи (эти устройства называются защитными реле ).

    Поскольку автоматические выключатели не выходят из строя в условиях перегрузки по току — скорее, они просто размыкаются и могут быть повторно включены путем перемещения рычага — они с большей вероятностью будут обнаружены подключенными к цепи более длительным образом, чем предохранители. Фотография маленького автоматического выключателя представлена ​​здесь:

    Рисунок 1.40. Малый автоматический выключатель

    Снаружи он выглядит не более чем выключателем. Действительно, его можно было использовать как таковое. Однако его истинная функция — работать как устройство защиты от перегрузки по току.

    Следует отметить, что в некоторых автомобилях используются недорогие устройства, известные как плавкие вставки , для защиты от перегрузки по току в цепи зарядки аккумулятора из-за стоимости предохранителя и держателя соответствующего номинала. Плавкая вставка — это примитивный предохранитель, представляющий собой не что иное, как короткий кусок провода с резиновой изоляцией, предназначенный для плавления в случае перегрузки по току, без какой-либо твердой оболочки. Такие грубые и потенциально опасные устройства никогда не используются в промышленности или даже в жилых помещениях, в основном из-за встречающихся более высоких уровней напряжения и тока.По мнению автора, их применение даже в автомобильных схемах вызывает сомнения.

    Обозначение на электрической схеме для предохранителя представляет собой S-образную кривую:

    Рисунок 1.41 S-образная кривая

    Номинальные характеристики предохранителя

    Предохранители

    , как и следовало ожидать, в основном рассчитаны на ток: ампер. Хотя их работа зависит от самовыделения тепла в условиях чрезмерного тока за счет собственного электрического сопротивления предохранителя, они спроектированы так, чтобы вносить незначительное дополнительное сопротивление в цепи, которые они защищают.Это в значительной степени достигается за счет того, что плавкий провод делается как можно короче. Точно так же, как допустимая токовая нагрузка обычного провода не связана с его длиной (сплошной медный провод 10 калибра выдержит ток 40 ампер на открытом воздухе, независимо от длины или короткого отрезка), плавкий провод из определенного материала и калибра будет дуть при определенном токе независимо от того, как долго он длится. Поскольку длина не является фактором в текущем рейтинге, чем короче она может быть сделана, тем меньшее сопротивление будет между концом и концом.

    Однако разработчик предохранителя также должен учитывать, что происходит после сгорания предохранителя: оплавленные концы сплошного провода будут разделены воздушным зазором с полным напряжением питания между концами.Если предохранитель недостаточно длинный в цепи высокого напряжения, искра может перескочить с одного из концов расплавленного провода на другой, снова замкнув цепь:

    Рисунок 1.42 Принципиальная схема конструктора предохранителей Рисунок 1.43 Принципиальная схема конструктора предохранителей

    Следовательно, предохранители рассчитываются с точки зрения их допустимого напряжения, а также уровня тока, при котором они сработают.

    Некоторые большие промышленные предохранители имеют сменные проволочные элементы для снижения затрат. Корпус предохранителя представляет собой непрозрачный картридж многоразового использования, который защищает провод предохранителя от воздействия и защищает окружающие предметы от провода предохранителя.

    Номинальный ток предохранителя — это нечто большее, чем просто цифра. Если через предохранитель на 30 ампер пропускается ток в 35 ампер, он может внезапно перегореть или с задержкой перед перегоранием, в зависимости от других аспектов его конструкции. Некоторые предохранители предназначены для очень быстрого срабатывания, в то время как другие рассчитаны на более скромное время «открытия» или даже на замедленное срабатывание в зависимости от области применения. Последние предохранители иногда называют плавкими предохранителями с задержкой срабатывания из-за их преднамеренных характеристик задержки срабатывания.

    Классическим примером применения плавкого предохранителя с задержкой срабатывания является защита электродвигателя, где пусковых импульсов токов, в десять раз превышающих нормальный рабочий ток, обычно возникают каждый раз, когда двигатель запускается с полной остановки. Если бы в таком приложении использовались быстродействующие предохранители, двигатель никогда бы не запустился, потому что при нормальных уровнях пускового тока плавкий предохранитель (и) немедленно перегорел бы! Конструкция плавкого предохранителя с задержкой срабатывания такова, что элемент плавкого предохранителя имеет большую массу (но не большую допустимую нагрузку), чем эквивалентный быстродействующий плавкий предохранитель, что означает, что он будет нагреваться медленнее (но до той же конечной температуры) при любом заданном количестве. тока.

    На другом конце спектра действия предохранителей находятся так называемые полупроводниковые предохранители , предназначенные для очень быстрого размыкания в случае перегрузки по току. Полупроводниковые устройства, такие как транзисторы, обычно особенно нетерпимы к условиям перегрузки по току и поэтому требуют быстродействующей защиты от сверхтоков в мощных приложениях.

    Предохранители

    всегда должны размещаться на «горячей» стороне нагрузки в заземленных системах. Это сделано для того, чтобы нагрузка была полностью обесточена во всех отношениях после срабатывания предохранителя.Чтобы увидеть разницу между плавлением «горячей» стороны и «нейтральной» стороны нагрузки, сравните эти две схемы:

    Рисунок 1.44 Принципиальная схема конструктора предохранителей Рисунок 1.45 Принципиальная схема конструктора предохранителей

    В любом случае предохранитель успешно прервал ток в нагрузке, но нижняя цепь не может прервать потенциально опасное напряжение с обеих сторон нагрузки на землю, где может стоять человек. . Первая схема намного безопаснее.

    Как было сказано ранее, предохранители — не единственный используемый тип устройства защиты от сверхтоков.Устройства, похожие на выключатели, называемые автоматическими выключателями , часто (и чаще) используются для размыкания цепей с чрезмерным током, их популярность связана с тем, что они не разрушают себя в процессе размыкания цепи, как предохранители. В любом случае, размещение устройства защиты от перегрузки по току в цепи будет соответствовать тем же общим рекомендациям, перечисленным выше: а именно, «предохранить» сторону источника питания , а не , подключенную к земле.

    Хотя размещение защиты от перегрузки по току в цепи может определять относительную опасность поражения электрическим током в этой цепи при различных условиях, следует понимать, что такие устройства никогда не предназначались для защиты от поражения электрическим током.Ни предохранители, ни автоматические выключатели не предназначены для срабатывания в случае поражения электрическим током; скорее, они предназначены для открытия только в условиях потенциального перегрева проводника. Устройства максимального тока в первую очередь защищают проводники цепи от повреждения из-за перегрева (и опасности возгорания, связанной с чрезмерно горячими проводниками), и, во вторую очередь, защищают определенные части оборудования, такие как нагрузки и генераторы (некоторые быстродействующие предохранители предназначены для защиты особенно чувствительных электронных устройств. к скачкам тока).Поскольку уровни тока, необходимые для поражения электрическим током или поражения электрическим током, намного ниже, чем нормальные уровни тока обычных силовых нагрузок, состояние перегрузки по току не указывает на возникновение удара током. Существуют и другие устройства, предназначенные для обнаружения определенных условий удара (детекторы замыкания на землю являются наиболее популярными), но эти устройства строго служат этой единственной цели и не связаны с защитой проводов от перегрева.

    • Предохранитель представляет собой небольшой тонкий проводник, предназначенный для плавления и разделения на две части с целью размыкания цепи в случае чрезмерного тока.
    • Автоматический выключатель — это специально разработанный переключатель, который автоматически размыкается для прерывания тока цепи в случае перегрузки по току. Они могут срабатывать (размыкаться) термически, магнитными полями или внешними устройствами, называемыми «реле защиты», в зависимости от конструкции выключателя, его размера и области применения.
    • Предохранители
    • в первую очередь рассчитаны на максимальный ток, но также рассчитаны на то, какое падение напряжения они будут безопасно выдерживать после прерывания цепи.
    • Предохранители
    • могут быть сконструированы так, чтобы срабатывать быстро, медленно или где-то посередине при одинаковом максимальном уровне тока.
    • Лучшее место для установки предохранителя в заземленной энергосистеме — на пути незаземленного проводника к нагрузке. Таким образом, при перегорании предохранителя к нагрузке останется только заземленный (безопасный) провод, что сделает безопаснее для людей находиться рядом.

    Какой ток более опасен, переменный или постоянный?

    Бывали ли вы когда-нибудь поражение электрическим током? Что ж, многие люди думают, что постоянный ток более опасен, чем переменный, с точки зрения поражения электрическим током.Чтобы выяснить, какой из них более опасен, давайте вернемся к основам переменного и постоянного тока.

    Переменный ток

    Переменный ток течет в форме синусоиды (как показано на рисунке ниже). Ток переключает направление вперед и назад. Мы можем найти переменный ток в бытовой технике, свете, вентиляторах и т. Д.

    Постоянный ток

    Постоянный ток течет только в одном направлении (показано ниже). Его можно найти в электронных схемах, батареях и т. Д.

    Позвольте мне сказать вам, что, помимо вида тока, степень поражения электрическим током зависит также от других факторов. Например,

    Сумма текущих

    Например, поражение электрическим током переменным током от 15 до 20 миллиампер может быть чрезвычайно болезненным. Однако поражение электрическим током силой 100 миллиампер может привести к смерти.

    Путь тока

    Прохождение тока из правой руки в правую ногу может быть болезненным.Но когда он проходит через сердце из правой руки в левую, это может вызвать фибрилляцию желудочков. Это состояние обычно приводит к летальному исходу.

    Длительность

    Скажем, поражение электрическим током слабым током; 0,3 миллиампера также могут вызвать болезненный эффект, если держать его в течение более длительного времени.

    Сопротивление тела

    Сопротивление мокрого тела поражению электрическим током меньше по сравнению с сухим телом.

    Почему человеческое тело ощущает поражение электрическим током?

    Это довольно интересно узнать.Ток проникает внутрь тела через кожу. Внешний слой эпидермиса кожи состоит из белкового материала, называемого кератином.

    Кератин обеспечивает высочайшее сопротивление прохождению электричества. После слоя эпидермиса идут потовые железы, а затем кровеносные сосуды. Эти потовые железы и кровеносные сосуды состоят из различных ионов, которые являются хорошими проводниками электричества. Следовательно, кровеносные сосуды и потовые железы обладают низким сопротивлением прохождению электричества.

    Теперь возникает вопрос, как ток достигает тела и проходит через него.

    Отвечая на вышеупомянутый вопрос, внешний слой эпидермиса кожи действует как диэлектрик, внутренние потовые железы и ткани действуют как одна пластина конденсатора, а металлическая деталь, по которой проходит электрический ток, действует как другая пластина конденсатора. Из-за этого емкостного эффекта через тело проходит ток. Быстро меняющееся напряжение позволяет большему току проходить через тело.

    Какой ток более опасен: постоянный или переменный?

    Есть аргументы в пользу как переменного, так и постоянного тока.Эти аргументы основаны на экспериментах и ​​исследованиях, проведенных на людях (включая мужчин и женщин) и профессионалах отрасли, которые имеют опыт работы с обоими типами токов.

    Аргумент в пользу постоянного тока

    Жертвы, которые испытали поражение электрическим током постоянным током, говорят, что они не могут отдернуть руку, потому что постоянный ток течет постоянно. Этот эффект похож на электрический дверной звонок, питаемый постоянным током. Следовательно, считается, что удар постоянным током более опасен.

    Принимая во внимание, что в случае переменного тока человек, испытывающий поражение электрическим током, может отдернуть руку, когда ток упадет до нуля. Следовательно, считается, что электрический ток переменного тока менее опасен, чем постоянный ток.

    Аргумент в пользу переменного тока

    Когда человек переживает поражение электрическим током, его цель — избавиться от него и спасти жизнь. Неизвестно, что происходит внутри мышц.

    Согласно экспериментам Чарльза Далзиэля на мужчинах и женщинах, сокращение мышц происходит непрерывно в случае поражения электрическим током постоянным током.В то время как в случае переменного тока человек, испытывающий электрический шок, подвергается серии сокращений мышц. Серия сокращений мышц вызывает очень серьезные повреждения мышц.

    Из-за емкостного поведения кожи, контактирующей с проводником с током, через тело может пройти больше тока, если напряжение быстро меняется. Исследования показали, что двукратное увеличение напряжения увеличивает семикратное увеличение тока.

    Порог «отпускания» постоянного тока выше порогового значения «отпускания» переменного тока.Для получения эффекта, аналогичного действию переменного тока, требуется больше постоянного тока.

    Эти аргументы основаны не только на экспериментах, проводимых на мужчинах и женщинах, но также были изучены с медицинской точки зрения. Следовательно, аргумент в пользу переменного тока верен.

    Теперь можно резюмировать, что переменный ток более опасен, чем постоянный. Что ж, не следует бояться электричества, но следует помнить, что и переменный ток, и постоянный ток могут быть опасны для человеческого тела, и при работе с любым из них необходимо учитывать меры безопасности.

    Список литературы

    • W. B. Kouwenhoven и O. R. Langworthy, «Эффекты электрического удара-II». Транзакция IEEE (A.I.E.E.).
    • Чарльз Ф. Далзил, «Воздействие электрического шока на человека».
    • В. Б. Кувенховен и Д. Р. Хукер, «Частотные эффекты поражения электрическим током».
    • Джон Кадик, «Справочник по электробезопасности», 3-е издание, Макгроу Хилл.
    • Раймонд М. Фиш и Лесли А. Геддес, «Проведение электрического тока через человеческое тело», Журнал открытого доступа по пластической хирургии.
    • Чарльз Ф. Далзил и Эрик Одген, «Влияние частоты на отпускаемые токи», IEEE Transaction.
    • Марк В. Кролл и Дорин Панеску, «Физика поражения электрическим током», Springer Science + Business Media New York, 2012.

    Крунал Шах — увлеченный педагог и консультант по вопросам карьеры с опытом работы в качестве предпринимателя. В настоящее время он работает директором Subodh Tech Private Limited, где занимается организацией профессионального обучения и консультированием по инженерным вопросам.

    Добавить комментарий

    Ваш адрес email не будет опубликован. Обязательные поля помечены *